Physio exam 3

Ace your homework & exams now with Quizwiz!

Anatomy of Nephrons

-Cortical and Juxtamedullary Nephrons -Smallest functioning unit of the kidney bc they can help produce urine. - Glomarulis (provides blood supply to allow for filtration of molecules to occure) -bowmans capsule (surrounds glomarulis) <Renal Corpussle> -Proximal Conviluted Tubule -Loop of henley ~thin descending limb ~thin ascending limb ~thick ascending limb -Distal convoluted tubule (DCT) -collecting duct

Proximal Convoluted tubule

-Cuboidal epithelial cells (contain mitochondria that can help produce ATP and energy that help allow for active trnsport of sodium back into blood stream and microvilli that help increase surface area and reobsorbtion) -Majority of reobsorbation happens here (sodium) *When sodium is absorbed so is water -blood glucose between 70-100mg/dl is normal

Major Anatomy of the kidney

-Fibrous capsule (along outside and helps maintain structure of kidney), -Renal cortex (where urine production occurs) <Renal Lobe> -Renal Medulla/Pyramid - Renal Pappila (Apex of pyramid/medulla) - Renal column (in between the pyramids, and maintain the integrity and structure of kidney) -Minor calyx (minor callicies) -Major callyx (major cllicies) -Renal pelvis -Hilus (indentation that can contain blood supply and innervation)

1. The answer is D. Distal K+ secretion is decreased by factors that decrease the driving force for passive diffusion of K+ across the luminal membrane. Because spironolactone is an aldosterone antagonist, it reduces K+ secretion. Alkalosis, a diet high in K+, and hyperaldosteronism all increase [K+] in the distal cells and thereby increase K+ secretion. Thiazide diuretics increase flow through the distal tubule and dilute the luminal [K+] so that the driving force for K+ secretion is increased.

1. Secretion of K+ by the distal tubule will be decreased by(A) metabolic alkalosis(B) a high-K+ diet(C) hyperaldosteronism(D) spironolactone administration(E) thiazide diuretic administration

Skin turns yellow when bilirubin levels exceed _____mg/dl.

1.5

12. The answer is A. The Henderson-Hasselbalch equation can be used to calculate the ratio of HA/A-:7.4 = 5.4 + log(A-/HA)

12. A buffer pair (HA/A-) has a pK of 5.4. At a blood pH of 7.4, the concentration of HA is(A) 1/100 that of A-(B) 1/10 that of A-(C) equal to that of A-(D) 10 times that of A-(E) 100 times that of A-

Lifespan of erythrocytes

120 days

What is the range of BPM for ventricular tachycardia?

120-250 BPM

Final urine is

1200 mOsm/L

24. The answer is A. Para-aminohippuric acid (PAH) has the greatest clearance of all of the substances because it is both filtered and secreted. Inulin is only filtered. The other substances are filtered and subsequently reabsorbed; therefore, they will have clearances that are lower than the inulin clearance.

24. Which of the following substances has the highest renal clearance?(A) Para-aminohippuric acid (PAH)(B) Inulin(C) Glucose(D) Na+(E) Cl-

How much blood does GI receive

25%

How much blood does muscle receive

25%

How much blood does the renal receive

25%

What is the percentage of blood that goes to the renal system?

25% goes into kidney

34. The answer is A. The history of vomiting (in the absence of any other information) indicates loss of gastric H+ and, as a result, metabolic alkalosis (with respiratory compensation).

34. Which set of arterial blood values describes a patient with a 5-day history of vomiting?

35. The answer is E. K+ is secreted by the late distal tubule and collecting ducts. Because this secretion is affected by dietary K+, a person who is on a high-K+ diet can secrete more K+ into the urine than was originally filtered. At all of the other nephron sites, the amount of K+ in the tubular fluid is either equal to the amount filtered (site A) or less than the amount filtered (because K+ is reabsorbed in the proximal tubule and the loop of Henle).

35. At which nephron site does the amount of K+ in tubular fluid exceed the amount of filtered K+ in a person on a high-K+ diet?(A) Site A(B) Site B(C) Site C(D) Site D(E) Site E

Filtered HCO3 Regulation

-Filtered load: Increases in filtered load result in increased rates of reabsorption. Plasma HCO3 concentration becomes very high (metabolic alkalosis). Will reach capacity and excrete HCO3 into the urine. -Pco2: Increases will result in increased HCO3 reabsorption because intracellular H+ for secretion increased. Decreases will result in decreased HCO3 reabsorption because H+ supply is decreased. These are the physiologic basis for the renal compensation for respiratory acidosis and alkalosis. -ECF volume: Expansion results in decrease HCO3. Concentration (contraction alkalosis) results in increase. -Angiotensin 2: Stimulates sodium hydrogen exchange, thus increasing HCO3 reabsorption. Contributes to contraction alkalosis that occurs secondary to ECF volume contraction

Mechanism of Urine production

-Filtration: a filtrate of the blood leaves the kidney capillaries and enters renal tubule (occurs at glomarulis) -Reobsorbtion: Most nutrients, water and essential ions are recovered from the filtrate and returned to the blood. -Secretion: Certain substances secreted from the blood into the filtrate product to be eliminated. -Excretion: Process of eliminating or expelling waste matter through the final excretory product, urine.

Fixed H+ excretion

-Fixed H+ produced from the catabolism of protein and phospholipid is excreted by two mechanisms: , titratable acid and NH4+. -Excretion as titratable acid (H2PO4): Depends on the amount of urinary buffer present and the pK of the buffer. -The H+ is secreted into the lumen by an H+-ATPase, and the HCO3- is reabsorbed into the blood (new HCO3). In the urine, the secreted H+ combines with filtered HPO4-2 to form H2PO4-, which is excreted as titratable acid -As a result of H+ secretion, the pH of urine becomes progressively lower. The minimum urinary pH is 4.4

Glycouria

-Glucose in the urine

Duration of QRS complex?

0.06 - 0.1 second

Duration of PR interval?

0.1 - 0.2 seconds

Normal duration for AV node is

0.1 - 0.2 seconds

Duration of QT interval

0.2 - 0.4 seconds

the first heart sound corresponds to poin

1

Treatment for Leukemia

1) Start with chemotherapy 2) Continue with interferon-alpha (INFa), which slows down production of immature leukocytes 3) Radiation therapy kills and destroys cancer cells 4) Last stage is stem cell transplantation

Steps in cycle

1-2: Isovolumetric contraction 2-3: Ventricular ejection 3-4: Isovolumetric relaxation 4-1: Ventricular filling

36. The answer is D. A person who is deprived of water will have high circulating levels of antidiuretic hormone (ADH). The tubular fluid/plasma (TF/P) osmolarity is 1.0 throughout the proximal tubule, regardless of ADH status. In antidiuresis, TF/P osmolarity > 1.0 at site C because of equilibration of the tubular fluid with the large corticopapillary osmotic gradient. At site E, TF/P osmolarity > 1.0 because of water reabsorption out of the collecting ducts and equilibration with the corticopapillary gradient. At site D, the tubular fluid is diluted because NaCl is reabsorbed in the thick ascending limb without water, making TF/P osmolarity < 1.0.

36. At which nephron site is the tubular fluid/plasma (TF/P) osmolarity lowest in a person who has been deprived of water?(A) Site A(B) Site B(C) Site C(D) Site D(E) Site E

37. The answer is E. Because inulin, once filtered, is neither reabsorbed nor secreted, its concentration in tubular fluid reflects the amount of water remaining in the tubule. In antidiuresis, water is reabsorbed throughout the nephron (except in the thick ascending limb and cortical diluting segment). Thus, inulin concentration in the tubular fluid progressively rises along the nephron as water is reabsorbed, and will be highest in the final urine.

37. At which nephron site is the tubular fluid inulin concentration highest during antidiuresis?(A) Site A(B) Site B(C) Site C(D) Site D(E) Site E

38. The answer is A. The tubular fluid inulin concentration depends on the amount of water present. As water reabsorption occurs along the nephron, the inulin concentration progressively increases. Thus, the tubular fluid inulin concentration is lowest in Bowman's space, prior to any water reabsorption. 38. At which nephron site is the tubular fluid inulin concentration lowest?(A) Site A(B) Site B(C) Site C(D) Site D(E) Site E

38. At which nephron site is the tubular fluid inulin concentration lowest?(A) Site A(B) Site B(C) Site C(D) Site D(E) Site E

39. The answer is A. Glucose is extensively reabsorbed in the early proximal tubule by the Na+-glucose cotransporter. The glucose concentration in tubular fluid is highest in Bowman's space before any reabsorption has occurred.

39. At which nephron site is the tubular fluid glucose concentration highest?(A) Site A(B) Site B(C) Site C(D) Site D(E) Site E

Pressure in large veins is what?

4 mmHg

How many steps does the cardiac cycle have?

4 steps?

Women normally have how much blood in the body?

4-5 Liters

Phosphate (renal regulation)

85% is reabsorbed through proximal tubule by sodium phosphate cotransport. 15% is filtered load in excreted urine. Parathyroid hormone inhibits phosphate reabsorption in proximal tubule by activating adenylate cyclase, causing phosphaturia and increased urinary cAMP.

9. The answer is C. Mannitol is a marker substance for the extracellular fluid (ECF) volume. ECF volume = amount of mannitol/concentration of mannitol = 1 g - 0.2 g/0.08 g/L = 10 L.

9. One gram of mannitol was injected into a woman. After equilibration, a plasma sample had a mannitol concentration of 0.08 g/L. During the equilibration period, 20% of the injected mannitol was excreted in the urine. The subject's(A) extracellular fluid volume is 1 L(B) intracellular fluid volume is 1 L(C) ECF volume is 10 L(D) ICF volume is 10 L(E) interstitial volume is 12.5 L

How much of plasma is water?

90%

Where is Eythropoietin secreted. What is it used for?

90% by kidney and 10% by liver Stimulates bone marrow for production of RBCs

Pyelonephritis

90% is by E. coli Treatment: Antibiotics Bacterial infection signs/symptoms: Back pain Abdominal pain Vomiting Fever Bacteria itself can change urine smell and color

a hospitalized patient has an ejection fraction of .4, a heart rate of 95 bpm, and a cardiac output of 3.5 L/min what is the patient's end-diastole volume?

92 mL

What percentage of pulmonary artery contents is released into aorta through ductus arteriosus ?

98

First degree AV node blocks duration is

<0.2 seconds

Basophils

A circulating leukocyte that produces histamine. Rare in blood stream. Allergic reactions. Inflammation

Metabolic acidosis

Acidity of the blood. Overproduction or ingestion of fixed-acid or loss of base products in arterial hydrogen (acidemia). H2SO4, by product of protein catabolism, H3PO4: phospholipid catablosim, ketoacids, lactic acid, salicylic acid. Bicarbonate HCO3 acts as a buffer to relieve blood of excess fixed acid (becomes H2SO3). Loss of buffer will continue acidemia, respiratory system will try to compensate causing hyperventilation "Kussmaul breathing". Renal system will attempt to compensate by excreting increased amount of fixed acids in titratable forms an ammonium by-product, and reabsorbing new formed HCO3 bicarbonate to buffer.

CCk acts on which types of pancreatic cells?

Acinar

The ____ is lined with ____ cells and secretes initial saliva.

Acinus, acinar

What are eosinophils for?

Active during allergic reaction

A patient complains of pains in the right hypochondriac and and epigastric regions and has recently lost 50+ pounds within the past couple of months. What is the correct diagnosis?

Acute cholecystitis

What are the glands involved in secondary hypertension?

Adrenal cortex gland, and medulla, as well as thyroid gland and parathyroid gland

Dromotropic effects

Affecting the heart's velocity of conduction. Changes in conduction velocity. Primarily in the AV node. -A negative effect decreases conduction velocity through the AV node, slowing the conduction of action potentials from the atria to the ventricles and increasing PR interval -A positive effect increases conduction velocity through the AV node, speeding the conduction of action potentials from the atria to the ventricles and decreasing PR interval.

__________ fibers carry sensory information from chemoreceptors and mechanoreceptors in the GI tract to the brain stem and spinal cord.

Afferent

What are the 2 types of Leukocytes?

Agranular and granular

Blood carrys what hormone to the kidneys

Aldosterone

Na+, Cl-, and H2O absorption in collecting tubule depend on what?

Aldosterone

One part of receptor on nephron is for...

Aldosterone

What hormones are secreted by adrenal cortex?

Aldosterone, cortisol, androgen

Metabolic alkalosis

Alkalinity of the blood "alkalemia". Excessive loss of fixed acid or increase in alkaline by product. May be due to hyperaldosteronism, where excess aldosterone increases absorption of HCO3 bicarbonate into the blood. Hypoaldosteronism: deficiency of aldosterone, resulting in increased absorption of H+ into the blood. -Arterial bicarbonate concentration increases -Alklaemia causes a compensatory decrease in respiration rate -Renal system compensates by increasing urine excretion. -Vomiting means loss of H+, which causes an increase in the reabsorption of HCO3 bicarbonate. Increases blood alkalinity, worsening condition.

Which valves are opened or closed during isometric contraction?

All valves are closed (For this reason, there is no change in volume of blood)

Third degree AV node block

Almost complete interruption (90%) between atrium and ventricle (3 successive P waves and then QRS)

What does globulins contain?

Alpha-1, Alpha-2, Beta, and Gamma

Bicuspid valve

Also known as mitral valve. Left atrioventricular valves. Separates left atrium and left ventricle. Lets oxygen-rich (oxygenated) blood pass from left atrium into left ventricle.

Cardiac output

Amount of blood each ventricle can pump in one minute. At rest, output is roughly 5 liters of blood every minute. During exercise, can increase up to 20 liters per minute (normal individual). An athlete: 35-40 liters. CO= stroke volume * heart rate

Two antibiotics used to kill H. pylori are ______ and ______.

Amoxicillin, tetracycline

Superficial to the major duodenal papilla where the common bile duct and pancreatic duct meet, the ______ ___ _______ can be found.

Ampulla of Vater

Heart murmur

An abnormal sound from the heart produced by defects in the chambers or valves. Generated by turbulent blood flow. May occur inside or outside heart. May be physiological (benign) or pathological (abnormal) Causes: Stenosis restricting opening of valve, causing turbulence, valve insufficiency (regurgitation); allowing backflow of blood when valve is supposed to be closed.

Deficiency of any of these factors can lead to what?

Anemia (reduction of rbc)

Normocytic anemia

Anemia due to blood loss (blood volume loss, not specifically erythrocytes) as a direct result of trauma. Ex: child birth

Macrocytic anemia

Anemia due to vitamin B12 deficiency or Folic acid (folate) deficiency. Resulting in decrease in erythrocytes specifically. -If only B12 is deficient, the condition is called pernicious anemia.

High blood pressure drugs

Angiotensin-converting enzyme (ACE) inhibitors: Captopril, Ramipril Angiotensin || receptor blockers (ARBs) : Valsartan Diuretics: Thiazide diuretics such as hydrochlorothiazide Calcium channel blockers: Felodipine, Benidipine Beta-1-adrenergic blocking agents: Propranolol or Atenolol

What part of the stomach is this?

Angular notch/incisure

___________ neurons elease POMC in the hypothalamic centers and cause decreased appetite.

Anorexigenic

_______ drugs inhibit production of saliva.

Anticholinergic

What part of the stomach is highlighted?

Antrum

The endocrine cells of the mucous membrane in the stomach are mainly found in the _____ and secrete _____, _____, _____, and ______.

Antrum, serotonin, gastrin, histamine, somatostatin

What are some other signs of myocardial infarction?

Anxiety Excess sweating Palpitations Respiratory disorder Vertigo EKG shows either elevated or depressed ST segment High positive R Large, negative Q waves Blood test shows high level of lactic acid (increase of enzyme called lactic dehydrogenase) Increased troponin T and I Increased myoglobin

Symptoms of glomerulonephritis

Any factor that leads to obstruction/destruction of any organ leads to hypertension Because of bacterial infection, autoimmune, etc. then there is low filtration rate Severe edema Hypertension Puffy face, eyes, legs Because fluid remains in body instead of excreted as urine In the case of bacterial infection, patient has fever, kidney pain (back pain), abdominal pain, radiational pain in abdominal cavity

Oxygenated to Deoxygenated blood

Aorta takes oxygenated blood to entire body and distributes into tissues through capillaries, arterioles and arteries, which absorb oxygen. The now deoxygenated blood is collected by venules from where it flows into veins. Veins take the blood to the inferior and superior vena cava, which return the blood to the heart (right atrium).

A patient comes to you complaining of constipation and pain in the right upper quadrant of their abdomen. The correct diagnosis is what?

Appendicitis

What is the normal range for Ca2+

Around 10mg/dL

What is bigger in diameter, arteries or capillaries?

Arteries

Where is the site of highest resistance in cardiovascular system?

Arteriloes

A patient with gastritis shows no sign of H. pylori infection, but has severe joint pain and takes medication for it. What is the most likely etiology of this case of gastritis?

Aspirin and other anti-inflammatory drugs

What medication can you use to manage myocardial infarction?

Aspirine and oxygen therapy

7. The answer is B. To answer this question, calculate the glomerular filtration rate (GFR) and Cx. GFR = 150 mg/mL × 1 mL/min ÷ 1 mg/mL = 150 mL/min. Cx = 100 mg/mL × 1 mL/min ÷ 2 mg/mL = 50 mL/min. Because the clearance of X is less than the clearance of inulin (or GFR), net reabsorption of X must have occurred. Clearance data alone cannot determine whether there has also been secretion of X. Because GFR cannot be measured with a substance that is reabsorbed, X would not be suitable.

Assuming that X is freely filtered, which of the following statements is most correct?(A) There is net secretion of X(B) There is net reabsorption of X(C) There is both reabsorption and secretion of X(D) The clearance of X could be used to measure the glomerular filtration rate (GFR) (E) The clearance of X is greater than the clearance of inulin

Step 3-4: Isovolumetric relaxation

At point 3, ventricle relaxes. When ventricular pressure decreases to less than aortic pressure, aortic valve closes. Ventricular volume is now constant.

Where is the site of gas and nutrient exchange within blood?

At the capilaries

5. The answer is C. Glomerular filtration will stop when the net ultrafiltration pressure across the glomerular capillary is zero; that is, when the force that favors filtration (47 mm Hg) exactly equals the forces that oppose filtration (10 mm Hg + 37 mm Hg).

At what value of glomerular capillary oncotic pressure would glomerular filtration stop? (A) 57 mm Hg(B) 47 mm Hg(C) 37 mm Hg(D) 10 mm Hg(E) 0 mm Hg

P wave

Atrial depolarization. Spreads from SA node to throughout the atria. Usually 0.08 seconds to 0.1 seconds long.

Signs and symptoms of mitral stenosis

Atrial fibrillation or flutter Frequent P waves in EKG Pulmonary edema and hypertension Risk for coagulation and formation of blood clot Hypertension

Having frequent P waves on EKG can show what heart problem?

Atrial flutter and fibrillation

Stretch receptors will stimulate secretion of what?

Atrial natriuretic peptide (ANP) from atrial tissue

Where are stretch receptors located?

Atrium of the heart

T cells

Attack all foreign cells

Control of heart rate

Autonomic Nervous System. Powerful control over heart rate and force of contraction. Heart is innervated by both parasympathetic (mainly SA and AV nodes, little strength), which will decrease heart rate, force of contraction and sympathetic nervous system (SA and AV node, much strength) which will increase heart rate and force of contraction. *If removed, the heart would beat at its own natural rhythm of roughly 100 bpm. Yet, the resting heart rate of a normal individual is roughly 70 bpm. An individual at rest, hasconstant activity from the parasympathetic system keeping the heart rate slowed to roughly 70 bpm

What are the 3 types of Agranular cells

B and T lymphocytes and Monocyte

What is Macrocytic anemia?

B12 and Folic acid deficiency

Glomerulonephritis

Bacteria causes infection in glomerular part of Bowman's capsule and decreases the filtration rate

What is the first sign of myocardial infarction?

Be wary of duration and location of pain -Duration : longer than 20 minutes -Location : Retrosternal pain, left shoulder, neck area (submandibular region), left shoulder, posterior region of left shoulder, left arm, left forearm, last two fingers (ring and pinky)

Why do we use digoxin in mitral stenosis?

Because digoxin has a negative effect on chronotropic(heart rate) and a positive effect on inotropic (contractility of myocardium)

Why can you use aspirine to manage myocardial infarction?

Because it dilates the blood vessels and is a anticoagulant

Why are there high levels of myoglobin during myocardial infarction?

Because its levels increase during low oxygen levels

Why can the blood flow easily through venous system?

Because of the position of the valves inside the vein

How do we have blood flow in large blood vessels such as superior/inferior vena cava?

Because of valves and contraction of skeletal muscle

Why is there an increase in Tropinin T and I during myocardial infarction?

Because troponin I shows there is something wrong with muscle contraction

What is the function of androgen in females?

Becomes active during puberty, which controls first signs of puberty

QT interval

Beginning of Q wave to beginning of T wave. Represents the time for both ventricular depolarization and repolarization to occur. Roughly estimates the ventricular action potential. 0.2 seconds to 0.4 seconds depending on heart rate. High heart rates: Ventricular action potentials shorten, which decreases the QT interval. Can be diagnostic to certain types of tachyarrythmias.

Absolute Refractory period (ARP)

Begins with upstroke of action potential and ends after plateau. No action potential can be initiated.

Block Ca2+ channels using Ca2+ blocker

Benidipine

ECG electrode lead 3

Between left arm and left leg

Pulmonary circulation

Between lungs and heart. Takes deoxygenated blood from heart to lungs and returns in back to the heart. Pulmonary artery takes blood from heart to lungs. Pulmonary veins take blood from lungs to heart.

Systemic circulation

Between organs (body) and heart. Superior and inferior vena cava take blood from body to heart. Aorta takes blood from heart to different parts of body. Carries oxygenated blood away from the heart and returns deoxygenated blood to heart. Pressure decreases progressively as blood flows through because of resistance to blood flow. Pressure is highest in aorta and large arteries and lowest in vena cava. Pressure: -Aorta, 100 mm Hg -Arterioles, 50 mm Hg -Capillaries, 20 mm Hg -Vena cava, 4 mm Hg

ECG electrodes lead 1

Between right arm and left arm

ECG electrodes lead 2

Between right arm and left leg

Secretin causes an increase in secretion of biliary and pancreatic _________, while decreasing gastric secretion of _______.

Bicarbonate, H+

Bile contains _____ _____, ______, ______, and ______ _____ (______).

Bile salts, cholesterol, phospholipids, bile pigements, bilirubin

_______ is a heme metabolite and is greenish in color.

Bilirubin

An antrectomy and vagotomy are characteristic of both forms of the ______ procedure.

Billroth

Atrial natriuretic peptide (ANP)

Binds to set of specific receptors. Receptor-agonist binding causes a reduction in blood volume, cardiac output and systemic blood pressure. Inhibits renin-secretion, thus inhibiting the renin-angiotensin system. Reduces aldosterone secretion. Relaxes vascular smooth muscle in arterioles and venules.

Coarctation of the aorta

Birth defect of congenital heart condition in which a portion of the Aortic artery walls are narrowed. Symptoms: Hypertension, tension in head and neck, damage or rupture of cerebral arteries can occur resulting in intracranial hemorrhage, increased pulse in the cartoid artery, low pulse in femoral artery (if patient has turner syndrome XO monosomy) Treatment: Surgical removal of narrowed portion of the aorta

What is the treatment for ventricular extrasystole if there is excess Na+?

Block Na+ channels with Pheytonin

What is the treatment for ventricular extrasystole if there excess beta-1?

Block beta-1 with propranolol

G cells secrete gastrin into the ______.

Blood

What carries ADH?

Blood

Thrombosis

Blood clot formation as a result of abnormal coagulation, resulting in increased risk for myocardial infarctions and stroke in the cerebral arteries.

What are the function of platelets?

Blood clotting

What are thrombocytes (platelets) for?

Blood clotting and secretes growth factors which are involved cell growth and cell proliferation

Thombocytes (Platelets)

Blood clotting. By releasing seratonin, which stimulates constriction of blood vessels, reducing flow of blood to injured area.

What is Normocytic anemia?

Blood loss due to heavy bleeding

An increase in either TPR or cardiac output increases what?

Blood pressure (hypertension)

Myocardial Infarction

Blood supply to certain areas of the myocardium is obstructed. Causes: atherosclerosis, stress, male gender, diabetes, family history of coronary artery disease (genetic or hereditary factors), high blood pressure, smoking, unhealthy cholesterol levels (high LDL, low HDL), chronic kidney disease. Signs and symptoms: chest pain (angina pectoris), pain in only one part of body (may move from chest to arms, shoulder, neck, teeth, jaw, belly, back). Can be severe or mild pain. Tight band around chest, bad indigestion, heavy feeling on chest, squeezing or heavy pressure. Usually lasting longer than 20 minutes. Sweating, anxiety, cough, fainting, dizziness, nausea, vomiting, palpitations (fast heart beat), dyspnea. Diagnosis: Ischemic type chest pain lasting longer than 20 minutes. Changes in ECG serial tracings. Rise and fall of serum cardiac biomarkers (creatine kinase: MB fraction, troponin T/I, myoglobin, lactate dehydrogenase). High positive R, large negative Q waves, ST segment elevation or depression, coronary intervention. Treatment: Medical emergency requiring immediate attention: oxygen, aspirin, nitroglycerin.

What part of the stomach is highlighted?

Body

Obstruction/infection of lymphatic system will lead to accumulation of

Body fluid and cause edema

After birth, where does blood stem cell production take place?

Bone marrow

Where is the problem located in primary polyvera?

Bone marrow(irritation or overreaction)

Cortisol is for

Breaks down protein Maintains blood glucose levels Excess cortisol causes central obesity Stimulates/increases systolic and diastolic pressures Increases total peripheral resistance (TPR)

Phase 1 (ventricles, atria, purkinje)

Brief period of initial repolarization. Caused by outward current because of movement of sodium ions out of the cell and in part because of decrease in sodium conductance.

The esophagus begins at which vertebra?

C6

Fats inhibit gastric emptying by stimulating the release of _____/

CCK

The duodenum secretes ______ in response to fatty chyme to induce gallbladder contraction.

CCK

This GI hormone potentiates secretin-induced stimulation of pancreatic bicarbonate secretion

CCK

This GI hormone stimulates growth of the exocrine pancreas

CCK

This GI hormone stimulates pancreatic enzyme secretion.

CCK

This hormone inhibits gastric emptying to allow more time for digestion and absorption.

CCK

This hormone stimulates the contraction of the gallbladder and relaxation of the sphincter of Oddi

CCK

_______ is homologous to gastrin.

CCK

Contraction of the gallbladder is due to the the hormone ______ or the neurotransmitter ______.

CCK, ACh

In response to fatty acids, amino acids, and small peptides, _______ is secreted by ___ cells of the duodenum.

CCK, I

myocardial contractility is best correlated with the intracellular concentration of

Ca2+

Magnesium is a natural

Ca2+ channel regulator

Which two ions play a role in slow wave production?

Calcium, Potassium

Which ion channels are involved in spike potentials?

Calcium, Sodium

What regulates the binding of calcium in smooth muscle cells of the GI tract?

Calmodulin

What are the results from increased afterload?

Decreased stoke volume and increased end-systolic volume

What is hypoxemia?

Deficiency of oxygen supply to myocardial tissue, which means there is failure of cell respiration and failure of ATP production

If we were to be under 180 L of blood, what condition are we considered?

Dehydrated and low blood pressure

Blood flow

Depends on: Total peripheral resistance (higher resistance means less flow). Pressure. Blood flows from high pressure to low pressure. Pressure gradient drives blood flow through.

Second half of P wave is

Depolarization and contraction of left atrium

First half of P wave is

Depolarization and contraction of right atrium

The cells in Ectopic foci have spontaneous

Depolarization and send signals to different parts of the heart

Na+ is important for

Depolarization of the cell

QRS complex

Depolarization of ventricles. Repolarization of atria. 0.06 seconds to 0.1 seconds. Occurs rapidly normally. *If prolonged, it is a problem within the ventricles. Can occur with bundle branch blocks, or whenever ventricular foci (abnormal pacemaker site) becomes the pacemaker driving the ventricle.

Phase 0 (ventricles, atria, purkinje)

Depolarization. Upstroke of action potential. Transient increase in sodium conductance. At peak of action potential, the membrane potential approaches the sodium equilibrium potential.

Capacitance (compliance)

Describes the distensibility of blood vessels. Inversely related to elastance or stiffness. The higher the amount of elastic tissue, the higher elastance, thus lower compliance. Directly proportional to volume and inversely proportional to pressure. Describes how volume changes in response to a change in pressure. Greater for veins than arteries. Decreases with age.

What causes aplastic anemia?

Destruction of normal structure of bone marrow.

Resting membrane potential

Determined by the conductance to potassium and approaches the potassium equilibrium potential. Ventricles, atria and purkinje fibers have stable resting membrane potentials (~90 millivolts) Inward current: brings positive charge into the cell and depolarizes the membrane potential. Outward current: takes positive charge outside of the cell and hyperpolarizes the membrane potential. *The role of Sodium, potassium-adenosine triphosphatase (ATPase) is to maintain ionic gradients across cell membranes

What are the causes of atrial flutter and fibrillation?

Diet pills Smoking Stress and anxiety Alcohol

K+ excretion depends on what 3 factors?

Diet, Aldosterone, and acidosis

The additive effect of the second messengers in the stimulation of parietal cells to secrete H+ means the second messenger for Gastrin-induced secretion must be ______.

Different

Which pathway (direct/indirect) uses neuronal action to stimulate parietal secretion?

Direct

Cardiac O2 consumption

Directly related to the amount of tension developed by ventricles. Increased by: increased afterload, size of heart, contractility, heart rate.

Kidney disease

Disease of kidneys. Cysts of the kidneys (polycystic kidney disease), tumors, renal failure, or obstructed renal artery (decrease renal filtration rate), resulting in fluid retention, build up, causing edema and increased blood pressure.

Do you know the cause of primary hypertension

Do not know the cause but we know some factors that can influence blood pressure

Secretin acts on which types of pancreatic cells?

Ductal

Supraventricular arrhythmia

Due to atrial or nodal extrasytole (heartbeat outside of normal rhythm) (ES). Sinus nodes often depolarize. -In atrial ES, the P wave is deformed but the QRS is normal. -In nodal ES the stimulation of atria is retrograde (moving backward), P wave is negative and is either masked by QRS wave or appears shortly after it. -Abnormal or ectopic (heterotopic) impulses may arise in the atria (atrial), in AV node, or in the ventricle (ventricular). The impulses from an atrial (ectopic) focus are transmitted to the ventricle, which is then thrown out of sinus rhythm.

What is the first factor of ventricular extrasystole?

Due to imbalance of electrolytes such as hypercalcemia that leads to extrasystole or arrhythmia

Chyme in the _____ causes the release of the hormonal mediators _______ and _______.

Duodenum, GIP, secretin

ECG Electrodes

ECG standard leads: three standard leads. Bipolar (detect change between two points). -Lead 1: Between right and left arm electrodes. Left arm is positive. -Lead 2: Between right arm and left leg. Left leg is positive. -Lead 3: Between left arm and left leg. Left leg is positive.

Gastric glands contain ______ cells that secrete serotonin and histamine.

ECL

Stroke volume equation

EDV-ESV (140-70=70)

Chronotropic effects

Effects of the ANS on heart rate. Produces changes in heart rate. -A negative effect decreases heart rate by decreasing the firing rate of the SA node. -A positive effect increases heart rate by increasing the firing rate of the SA node.

_________ fibers carry information from the brain stem and spinal cord to the GI tract.

Efferent

At the end of ventricular filling, what does it return to?

End diastolic volume (140 mL)

What is the amount of blood in isometric contraction?

End-diastolic volume, which is about 140 mL

In both patent ductus arteriosus and atrial septum defect, what kind of inflammation can occur?

Endocarditis

Inside heart, it is...

Endocardium, which is formed of papillary muscles

Neurocrine that stimulates contraction of GI smooth muscle and inhbits intestinal secretion of fluid and electrolytes.

Enkephalins

The duodenum secretes ______ which inhibits stomach peristalsis.

Enterogastrone

What are the 3 types of granular cells?

Eosinophils, basophils, and neutorphils

What hormones are secreted by the adrenal medulla?

Epinephrine and norepinephrine

Acid-Base balance

Equilibrium between acid and base concentrations in the body fluids. -Two types of acid produced: Volatile acid (CO2). Produced from aerobic metabolism of cells. Combines with H2O to form weak acid H2CO3 which dissociates into H+ and HCO3 by carbonic anhydrase: present in most cells, catalyzes the reversible action between CO2 and H2O. Non-volatile acids: Also called fixed-acids. Include: sulfuric acid H2SO4 and phosphoric acid. Normally produced at a rate of 40-60 moles/day. Some may be over produced in disease. These include: ketoacid, lactic acid, and salicylic acid.

What can reticulocyte convert to?

Erythrocyte

What are the 3 types of blood cells?

Erythrocytes (RBCs), Leukocytes, Platelets

Formed elements

Erythrocytes, leukocytes, platelets, blood cells.

Sometimes the kidney secretes what?

Erythropoeitin (stimulates bone marrow for production of RBCs)

When the distal end of the esophagus is closed.

Esophageal atresia

The ______ propels swallowed food into the stomach.

Esophagus

Aplastic anemia

Failure of blood cell production in the bone marrow. Abnormality in the bone marrow due to chemical or radiation damage. Inability to synthesize erythrocytes.

Deficiency of magnesium means

Failure to control Ca2+ channels

T/F: Atropine is capable of blocking ALL H+ secretion.

False

T/F: Duodenal ulcers are more uncommon than gastric ulcers.

False

T/F: Frequency of slow waves are influenced by hormonal and neuronal input.

False

T/F: Gastric juice is composed of just the exocrine secretions of the stomach glands.

False

T/F: Segemental contraction of the small intestine results in a net forward movement of chyme

False

T/F: Slow waves are not action potentials

False

T/F: The hepatic artery supplies the liver with 110ml of blood per minute.

False

T/F: The second messenger for gastrin mediated H+ secretion by parietal cells is the same as the second messengers used in vagal and histamine-based stimulation.

False

T/F: Troponin C regulates the binding of calcium in the smooth muscle of the GI tract

False

T/F: The contraction of only longitudinal muscle in the GI tract results in peristalsis.

Fasle

What is underneath pericardium?

Fat tissue and then myocardium

Phase 1

Few K+ ions leave the cell

What is Glomerular filtration rate for females vs males

Filtration rate is 150 mL/min in female; 125 mL/min in male

Glomerular Ultrafiltrate

Fluid that enters the glomerular capsule. Produces a large volume of filtrate. Filtered water must be returned or person would urinate to death.

Gastric secretion of gastric juice is stimulated by ______ _______.

Food ingestion

Blood pressure

Force exerted by circulating blood on the walls of blood vessels.

Glomerular filtration

Glomerular capillaries have large pores in walls and layer of Bowman's capsule in contact with glomerulus for filtration. Water with dissolved solutes can pass from blood plasma to inside of capsule and nephron tubules. Filtrate is produced by both kidneys per minute known as glomerular filtration rate (GFR) Averages: 115 mL/min in females, 125 mL/min in men. 180 L per day. GFR: Measurement of GFR-clearance of inulin. Inulin is filtered, but not reabsorbed or secreted by renal tubules. GFR= U inulin V/P inulin U= Urine concentration of inulin P= Plasma concentration of inulin

Healthy person cannot have what in there urine?

Glucose

Nephron structure can be destroyed by what?

Glucose

What can be symport from the 67% of Na+

Glucose Amino acid Phosphate Bicarbonate (HCO3-) Cl- K+

What is the treatment for ventricular extrasystole if there is excess Ca2+?

Go for Ca2+ channel blockers with verapamil

What is the treatment for ventricular extrasystole if there is excess K+?

Go for K+ channel blockers with sotalol

Gastric glands contain ______ cells which secrete mucous.

Goblet

The ______ the number of spike potentials the stronger the phasic contraction.

Greater

Frank-Sterling relationship

Greater venous return increases end-diastolic volume, which increases the length and tension to myocardium, and increases Ca2+ release, which causes strong muscle contraction → increases cardiac output (about 5 L/minute)

1st part of Frank-Sterling relationship

Greater venous return leads to increased cardiac output (amount of blood which is ejected into aorta per minute is approximately 5 L)

What antiports with Na+?

H+

_____ in the duodenum also inhibits gastric emptying.

H+

If vomiting occurs, ___ never is detected by the small intestine and therefore _____ is never delivered to the _____ resulting in metabolic ______.

H+, HCO3-, alkalosis

In the parietal cells, carbon dioxide and water are converted into ___ and _____ and this conversion is catalyzed by ____ ______.

H+, HCO3-, carbonic anhydrase

Duodenal ulcers are characterized by an increase in ____ secretion, higher than normal _____ levels after eating a meal, and an increase in parietal cell mass.

H+, gastrin

Histamine stimulates parietal secretion by binding to ______ receptors which are blocked by the drug ______.

H2, cimetidine

Ductal cells of the pancreas alter the initial secretion by adding ______ which is exchanged with _____ across the luminal membrane.

HCO3-, Cl-

Pepsinogen is a protein splitting enzyme that is activated by ______.

HCl

Parietal cells in the stomach produce ______ and the intrinsic factor for vitamin ______ absorption in the ileum.

HCl, B12

Signs of hypertension

Headache, vomiting, nausea Nose bleeding Vertigo Sleep disorder Palpitations Sweating

Signs/symptoms of hypertension

Headache, vomiting, sleep disorder, nose bleeding, vertigo, etc.

Sinus tachycardia

Heart rate is greater than 100 beats per minute. Normal Causes: Exercise, psychic excitation, fever, rise of 10 bpm

Sinus bradycardia

Heart rate is less than 60 beats per minute. Normal

Renal Clearance

Indicates the volume of plasma cleared of a substance per unit time. Units are mL/min and mL/24hr C=UV/P C= Clearance (mL) U= Urine concentration (mg) V= Urine volume (mL) P= Plasma concentration (mg)

Which pathway (direct/indirect) uses endocrine action to stimulate parietal secretion?

Indirect

What is endocarditis?

Infection and inflammation of the endocardium

Endocarditis

Inflammation of inner lining of the heart chambers and heart valves (endocardium). Usually a result of a blood infection (bacteria or other infectious agents, that can enter bloodstream during medical procedures). Bacteria can grow and may form infected clots that break off and travel to brain, lungs, kidneys, or spleen. Increase in risk: artificial heart valves, congential heart disease (atrial septal defect, patent ductus ateriosus), heart valve problems (ex: mitral insufficiency), history of rheumatic disease. Symptoms: abnormal urine color, chills, excessive sweating, fatigue, fever, joint pain, muscle aches/pains, night sweats, nail abnormalities (splinter hemorrhages under the nails), paleness. Testing: Blood culture and sensitivity (bacteria detection), chest x-ray, complete blood count (may show anemia), echocardiogram (heart ultrasound), erythrocyte sedimentation rate (ESR), transesophageal echocardiogram Treatment: long term high dose antibiotic treatment is needed (usually 4-6 weeks). Surgery to replace damaged heart valves.

Lupus

Inflammatory condition in which the immune system attacks the patient's own tissue.

What does alpha 1 and 2 globulin do?

Inhibits/Prevents activation of an enzyme called protease

What do you do with the insulin if you want to measure filtration rate?

Inject it because it will not be absorbed

Endocardium

Inner layer of the heart

GIP leads to increased _______ secretion and decreased ________.

Insulin, H+

Velocity of blood depends on what?

Intersectional area, which exists in capillary

The ______ _____ __ ______ serve as the pacemaker for the GI tract

Interstitial cells of Cajal

PR interval

Interval from beginning of P wave to beginning of Q wave (initial depolarization of ventricle). Varies with conduction velocity through AV node. Increases PR if nodal conduction decreases (heart block). 0.12 seconds to 0.20 seconds. Between the onset of atrial depolarization and ventricle depolarization. *If less than 0.2 seconds there is an AV conduction block. -Decreased (ex: increased conduction velocity through AV node) by stimulation of the sympathetic nervous system. -Increased (ex: decreased conduction velocity through AV node) by stimulation of the parasympathetic nervous system

What increases during ventricular ejection?

Intraventricular pressure

Contractility

Intrinsic ability of cardiac muscle to develop force at a given muscle length. Also called intropism. Related to the intracellular calcium concentration.

_____ _____ is a co-secretion product with _____, so destruction of parietal cells leads to _____ and ______.

Intrinsic factor, HCl, Pernicious anemia, achlorhydria

What is Microcytic anemia?

Iron deficiency

Microcytic anemia

Iron deficiency, anemia of chronic disease, thalasemia, sideroblastic anemia. Resulting in a decrease in erythrocytes specifically.

What do T lymphocytes do?

Is active and uptakes foreign body for degredation

What is the first step of the cardiac cycle?

Isometric contraction (1-2)

The rate of gastric emptying are fastest when the stomach contents are ______.

Isotonic

What is the third step of the cardiac cycle?

Isovolumetric relaxation

What does albumin absorb when present in blood vessels?

It absorbs extra extra fluid from the interstitial and intracellular space

What does ANP do?

It acts as a vasorelaxor (It relaxes blood vessels and lowers blood pressure)

What happens to the left atrium in mitral stenosis?

It causes of build up of blood which makes the atrium pump faster and faster

What does erythrocytes contain and carry? Lifespan?

It contains hemoglobin and each hemoglobin carries four oxygen molecules and contains iron Lifespan= 120 days

What does decreased renin mean for aldosterone?

It decreases as well

What is the pressure during isovolumetric relaxation?

It drops to 0 in ventricle

Why is fibrinogen important and where is it produced?

It is important for clotting factor and it is produced in the liver

What happens to gamma globulin when someone has hepatitis?

It protects and increases/activates immune cells against viruses

arteriography of a patient's left renal artery shows narrowing of the radius of the artery by 50%, what is the expected change in blood flow through the stenotic artery?

It will decrease to 1/16 of it's original flow.

Why is Ca2+ important for the conduction of the heart

Its for SA/AV node depolarization and myocardial contraction

A common sign of a damaged liver is _______.

Jaundice

Where can you hear tricuspid valve?

Junction of 5th rib with sternum on right side (5th intercostal space)

What does Na+ symport in the Henle tubule?

K+ and Cl-

Phase 4

K+ should reach -85 mV (Equilibrium potential for K+)

Periorbital edema

Kidney condition. Fluid is retained in the face and around the eyes (puffy face and eyes).

Erythropoietin

Kidney hormone that promotes red blood cell formation in bone marrow (hormone controlled)

These cells of the liver screen portal blood for bacteria and remove >99%.

Kupffer

Septum divides

L + R atria & L+ R Ventricles

When there is no oxygen this leads to formation of what?

Lactic acid

Why is lactic acid formed during myocardial infarction?

Lactic acid is formed due to oxygen deficiency

Capillaries

Largest total cross sectional and surface area. Consist of single layer of epithelial cells surrounded by basal lamina. Thin wall. Site of exchange of nutrients, water and gases. Total peripheral resistance is what capillaries are called. Pressure: 20 mmHG

Monocytes

Largest. Phagocytic cells. Kidney shaped nucleus. Mature to become macrophages. 4-8%

Where can you hear pulmonary valve?

Left 2nd intercostal space

What happens to blood flow during mitral stenosis?

Left atrium contracts and then it is unable to release contents into left ventricle

Secreted by fat cells, stimulates anorexigenic neurons and inhibits orexigenic neurons, decreasing appetite.

Leptin

What is hypercapnia?

Levels of CO2 is high in the blood

At the end of pregnancy, what does the ductus arteriosus turn into?

Ligamentum arteriosum

Chemoreceptors in cartoid/aortic bodies

Located near bifurcation of common cartoid arteries and aortic arch. High rates of O2 consumption and sensitive to decreases in partial pressure of oxygen (Po2). Decreases in Po2 activate vasomotor centers that produce vasoconstriction, an increase in TPR, and arterial pressure.

Mitral regurgiation

Long term disorder in which the heart's mitral valve does not close properly causing blood to flow backward (leak) into the upper heart chamber when the left lower chamber contracts. Condition is progressive. Causes: mitral valve prolapse, congential, atherosclerosis, endocarditis, heart tumors, high blood pressure, marfan syndrome, untreated syphillis. Symptoms: cough, fatigue, palpitations, shortness of breath during activity, urination (excessive at night), enlarged liver. Treatment: Depends on symptoms. Antibiotics reduce risk of infection (endocarditis: in patients who have mitral valve proplapse during dental work). Antihypertensive drugs and vasodilators, anticoagulant/antiplatelet (atrial fib). Digitalis to strengthen heartbeat, and diuretics to remove fluid from lungs.

Steroidal drugs

Long term use of synthetic cortisols (anti-inflammatory corticosteroids like prednisone) which are typically used to treat inflammatory auto-immune conditions like dermatitis or Crohns disease can lead to increases in: TPR, systolic/diastolic pressures, and increased arterial resistance.

Contraction causes shortening of a segment of the GI tract.

Longitudinal muscle

Anemia

Low level concentrations of erythrocyte or hemoglobin. Most commonly due to a deficiency in one of three conversion factors for converting reticulocytes into erythrocytes.

Heart sounds

Lub (S1). Dub (S2). 1st- AV valves close. 2nd- aortic and pulmonary valves close. First sound produced is the closure of AV valves (low pitch). Second sound is the closing of semilunar valves (high pitch). Third sound sometimes occur in middle of diastole, caused by blood flowing with rumbling motion into almost filled ventricles. Difficult to hear.

Atropine works at the ______ receptor, cimetidine works at the ______ receptor, and omeprazole works at the _______.

M3, H2, H+/K+-Pump

Intercalated discs

Maintain cell to cell cohesion

Androgen is

Male sex hormone but females have small amount

What are the Hematocrits for Males? Females?

Males= 42-52% Females= 37-47%

What is the blood volume in Males? Females?

Males= 5-6L Females= 4-5L

Atrial flutter

Many consecutuve atrial depolarizations, resulting in a quivering atrial heart muscle. Small "flutter waves" or saw tooth pattern (P waves before each QRS complex). Atria beat irregularly and faster than the ventricles. Causes: Different types of heart disease, stress, anxiety, caffeine, alcohol, tobacco, diet pills, surgery (open heart)

U wave

May be seen following the T wave. Represents the last remnants of ventricular repolarization. Inverted or prominent U waves indicate underlying pathology or conditions affecting repolarization.

The swallowing reflex is coordinated in the ______ by cranial nerves _____ and ______.

Medulla, 9, 10

The vomiting center in the _______ is stimulated by tickling the back of the throat, gastric _______, and _________ stimulation (motion sickness).

Medulla, distention, vestibular

Signs/symptoms of hypothyroidism

Mental and bone growth retardation Learning/memory disorders Motor and sensory disorders

How can a surgeon mess up during heart surgery to cause endocarditis?

Mess up on operation involving mitral valve

Serves to rid the upper GI tract of digestive secretions during fasting.

Migrating motor complex

What is the myocardium have inside it?

Mitochondria, and it has gap junctions and tight junctions

What valve is pushed on during ventricular filling?

Mitral and tricuspid valve, which release 70 mL of blood into ventricle

What is mitral regurgitation?

Mitral valve cannot close properly leading to blood getting into left atrium.

MEC are contractions that occur in 90m intervals to clear residual food out of the stomach and are mediated by _______.

Motilin

Which canidate hormone is involved in the interdigestive myoelectric complexes?

Motilin

What are the 3 candidate hormones?

Motilin, pancreatic polypeptide, GLP-1

Blood circulation

Movement of blood throughout the body. Starts at superior and inferior vena cava. Empty into right atrium (contain deoxygenated blood) Right atrium -> tricuspid valve -> right ventricle -> pulmonary valve -> pulmonary artery -> lungs -> pulmonary vein -> left atrium -> mitral valve -> left ventricle -> aorta -> body

which receptor mediates slowing of the heart?

Muscarinic receptors

Contraction of this layer causes a change in the surface area of the gi tract for secretion and absorption.

Muscularis mucosa

What are the 2 groups of blood stem cells

Myeloid and lymphoid stem cells

The _______ plexus primarily controls the motility of the GI smooth muscle.

Myenteric

Increase in platelet production can lead to what?

Myocardial infarction and stroke in CNS

Contraction of _____ cells ejects saliva into the mouth.

Myoepithelial

H. pylori converts urea to ______ through the use of ______.

NH3, urease

Acinar cells of the pancreas produce the initial secretion which is mostly composed of _____ and ______.

Na+, Cl-

Saliva has low ___ and ____ concentration and high ___ and ___ concentration.

Na+, Cl-, K+, HCO3-

Capillary then absorbs what minerals?

Na+, Cl-, and H2O

Where is the adrenal gland located?

Near the kidney

At one point, how low can the pressure be in isometric contraction?

Nearly 0

What is the pressure in the ventricle during ventricular filling?

Nearly 0

Parasympathetic has what kind of effect on cardiovascular system?

Negative

Intropic effects

Negative effect: decreases force of contraction Positive effect: increases force of contraction. When more action potentials occur more calcium enters the myocardial cells. During action potential plateaus, more calcium is released from SR. Greater tension is produced. Ex of positive effect: Increased heart rate: (1) Positive staircase: "Bowditch staircase" (or Treppe). Increased heart rate increases the force of contraction in a step-wise fashion as the intracellular calcium increases cumulatively. Postextrasystolic potentiation: the beat that occurs after an extrasystolic beat has increased force of contraction because extra calcium entered the cells. -Cardiac glycoides (digitalis)

What is the major part of the urinary production?

Nephrons

There are millions and millions of what in the kidneys?

Nephrons (Function is for production of urine)

What else can be used to help myocardial infarction?

Nitroglycerin since its also a anticoagulant

Will increasing the amount of WBCs mean you have a stronger immune system?

No because it depends on the type of WBC you havee

If blood glucose is above 400

No longer able to reabsorb, mainly excretion will occure.

Phase 1 and 2 (SA node)

Not present.

Systemic circulation

O2 blood taken away from heart to body CO2 blood from body taken back to heart

The stomach has a third layer of ______ fibers.

Oblique

A jaundiced patient's urine sample comes back negative for urobilin and positive for conjugated bilirubin. What is the correct diagnosis?

Obstructive jaundice

Acute Glomerulonephritis (AGN)

Often result of post-infection complications of the skin or throat via bacterial, parasitic, or viral pathogen (ex: strep, staph, pneumococcus, chicken pox, malaria). If non-infectious it may be due to membranoproliferative glomerulonephritis (overactive cell membrane proliferation thickens the porous tissue of glomerular vessels, decreasing and inhibiting filtrate outflow). Systemic lupus erythematosus or immunoglobulin A nephropathy (IgA nephropathy, "Berger's disease", most common)

You attempt to treat a patient with gastritis using Maalox, but the effect isn't potent enough to reduce the inflammation (H+ secretion is still too high to combat). The patient seems to have gastric ulcers as a result of NSAID use but not infection. Your next (and MOST EFFECTIVE) option for treatment would be _______ for ______ weeks.

Omeprazole, 8

Blood flow depends on what?

On blood pressure that exists in blood vessels and it depends on total peripheral resistance (TPR)

Step 4-1: Ventricular filling

Once left ventricle decreases to less than left atrial pressure, the mitral valve opens, filling the ventricle again. Ventricular volume increases to ~140 mL (end-diastolic volume)

Primary Glomerulonephritis

Only affects the kidneys with more localized symptoms

Phase 0 for SA and AV

Opening of Ca2+ channels (Ca2+ enters and causes depolarization of SA node cell)

Phase 3 for SA and AV

Opening of K+ channel (K+ leaves the cell)

Phase 4 for SA and AV

Opening of Na+ channels (Na enters the cell)

Phase 0

Opening of Na+ channels and Na+ inflow (-90 to +65 mV)

What region of the stomach is composed of the proximal body and fundus?

Orad region

___________ neurons release neuropeptide Y in the hypothalamic centers and stimulate appetite.

Orexigenic

Slow waves are ____ ______ _____ inherent to the smooth muscle cells of some parts of the GI tract

Oscillating membrane potentials

Epicardium (pericardium)

Outer layer of heart wall; "on the heart"; visceral layer of serous pericardium. Covers the heart

What is the cause of secondary polyvera?

Overproduction of erythropoietin(it is artificial/false)

What leads to Leukemia?

Overproduction of leukocytes (could be mature or immature)

When heart is active there is more consumption of what element?

Oxygen

What does the blood carry

Oxygen Carbon dioxide Vitamins Glucose Proteins Immune cells Hormones

First wave in EKG?

P wave

In EKG, for ectopic foci, you can see

P wave → QRS → T wave → P wave → QRS → abnormal QRS → abnormal QRS → abnormal QRS

What can you see more of on an EKG from the faster pumping of the atrium?

P waves

AV block

PR interval is prolonged. Slowing of impulse conduction. Damage to AV node. Three degrees: -First degree AV node block: PR interval exceeds .20 seconds. -Second degree AV node block: Occurs when AV node is damaged so severely that only one out of every two-four atrial electrical waves can pass through to the ventricles. P waves without associated QRS waves. -Third degree AV node block (complete AV node block): None of the atrial waves can pass through the AV node to the ventricles. Results in bradycardia.

Sinoatrial node (SA node)

Pacemaker of the heart. Unstable resting membrane potential. Exhibits phase 4 depolarization or automaticity. The AV node and His-Purkinje systems are latent pacemakers. that may exhibit automaticity and override SA node if it is suppressed. Located in the upper posterior wall of the right atrium. First area to spontaneously depolarize. Highly coordinated. -From SA node, travels to atrial muscle causing contraction, and then moves to ventricles.

The most important source of enzymes for digestion is the _______.

Pancreas

_________ ________ is the candidate hormone that inhibits pancreatic secretions.

Pancreatic polypeptide

Type of hiatal hernia where the cardia does not pass through the esophageal hiatus but part of the fundus and the peritoneum do. Not accompanied by regurgitation normally.

Paraesophageal hiatus hernia

Eosinophils

Parasitic worm infections. Turns off allergic responses. Bilobed.

What can have a negative effect on the cardiovascular system?

Parasympathetic by muscaniric receptors

Depolarization within the GI smooth muscle potential is a result of stimulation by the ________ nervous system, through the use of the neurotransmitter _________.

Parasympathetic, acetylcholine

The ____________ nervous system is usually excitatory on the functions of the GI tract.

Parasympthetic

Gastrin promotes the growth of which cell type found in the stomach?

Parietal

H+ is secreted by gastric ______ cells.

Parietal

Mumps is characterized by inflammation of the _____ gland.

Parotid

This nerve innervates the lower large intestine, rectum, and anus.

Pelvic

Chief cells of the stomach produce _______.

Pepsinogen

Per day how much blood is filtrated?

Per day we have 180 L of filtration (large amount of blood)

What is Hematocrit?

Percentage of blood volume that consists of red blood cells(RBCs)

Hematocrit

Percentage of blood volume that consists of red blood cells. Males: 42-52% Females: 37-47%

Heart is covered by membrane called?

Pericardium

______ ______ results from rapid and powerful response to an irritant in the GI tract.

Peristaltic rush

Which part of the renal artery division absorbs the most?

Peritubular capillary

Neutrophils

Phagocytic. Attacks bacterial cells. Multilobed. Most abundant.

What are neutrophils for?

Phagocytosis

What do Monocytes do?

Phagocytosis

While swallowing peristalsis begins in the _______.

Pharynx

The contractions that occur as a result of spike potentials are called _______ contractions.

Phasic

_________ contractions occur in th esophagus, gastric antrum, and small intestine, which contract and relax periodically.

Phasic

What kind of proteins do blood plasma have in them?

Plasma proteins (Albumin, Globulins, Fibrinogens)

Phase 2

Plateau (not clear if negative or positive because of opening of Ca2+ channels) Ca2+ enters ccell

Phase 2 (ventricles, atria, purkinje)

Plateau of action potential. Caused by transient increase in calcium conductance resulting in inward calcium current and by an increase in potassium conductance. Outward and inward currents are approximately equal

What do capillaries have instead of smooth muscle?

Pores, which is for gas exchange and nutrients

Abnormal QRS is a sign for

Postmyocardial infarction

What is ectopic foci

Postmyocardial infarction

When can normocytic anemia happen?

Pregnant female has heavy bleeding during delivery Amount of blood is less than normal

Gap junction

Present at intercalated discs. Low-resistance paths between cells for rapid spread of action potentials. Accounts for the observation that the heart behaves as an electrical syncytium.

Cardiac cycle: Period of ejection.

Pressure in ventricle must be greater than pressure in the aorta, for the blood to be ejected. When pressure in the left ventricle reach 80 mmHg, the aortic valve opens. Blood pours out of ventricles, pressure will continue to increase (~120 mL). Known as ejection period.

Is atrial pressure higher or lower than ventricular pressure during ventricular filling? Why?

Pressure is much higher Because atria receives venous return blood (about 70mL)

Blood pressure means the

Pressure that exists or comes to the wall of blood vessels by blood flow

Buffer

Prevent a change in pH when H+ ions are added or removed from a solution. Most effective within 1.0 pH. -Extracellular buffers: Major buffer is HCO3. Phosphate is a minor buffer, most important as a urinary buffer. H2PO4 is a tritratable buffer. -Intracellular buffer: Organic phosphates, proteins, hemoglobin (major buffer) *Physiologic pH range: deoxyhemoglobin is a better buffer than oxyhemoglobin.

Preganglionic sympathetic cholinergic fibers synapse in the ___________.

Prevertebral ganglia

Omeprazole is marketed under the name _______.

Prilosec

Two types of hypertention

Primary and secondary hypertension

What are the two types of polycythemia?

Primary or secondary polyvera

What two cells does the collecting tubule have?

Principal cell and alpha intercalating cells

B cells

Produce antibodies to attack again

Myelogenous luekemia

Produces too much immature + mature myelocytes

What 2 functions does the yolk sac have?

Production of blood stem cells and formation of germ cells

Non-steroidal drugs

Prolonged use can cause salt and water retention into the blood, increased sympathetic activity, increased blood pressure. Some birth controls use a combination of progesterone and estrogen which increase appetite, absorption of salt and fluid, resulting in edema, high blood pressure, weight gain and water weight gain.

Sometime, we need to block beta-1 receptor using beta-1 blocker

Propranolol

What are the two functional types of movement in the GI tract?

Propulsive, mixing

2 types of natural vasodilator

Prostaglandin, bradykinin

Myocardial cells/Nodal Conducting cells

Provide a self excitatory system for the heart to generate impulses and a transmission system for rapid conduction throughout the heart. Contract weakly. Contain myofibrils, surrounded by sarcoplasmic reticulum. Able to spontaneously generate action potentials without the help of neurons. Rapidly conduct action potentials to atrial and ventricular muscle. Two principle types: contractile cells (have similar features to skeletal muscle cells, contain actin and myosin) and nodal/conducting cells (similar to nerve cells) Action potential travels through the atria to the AV node then to bundle of His (AV bundle), and then to the purkinje fibers and then to the ventricular muscle.

Most colonic water absorption occurs in the ______ colon.

Proximal

Where is calcium mostly absorbed?

Proximal and distal tubules which is under control of parathyroid

Where is magnesium absorbed?

Proximal tubules

What is pulmonary circulation?

Pulmonary artery carries blood from heart to the lungs and pulmonary veins carry blood from lungs to the heart

What does the build up of fluid in the lungs lead to?

Pulmonary edema

When the pulmonary veins get hypertension what is it called?

Pulmonary hypertension

Signs/symptoms of pulmonary regurgitation

Pulmonary hypertension and edema Atrial fibrillation or flutter

Arterial pressure

Pulsatile. Not constant during cardiac cycle. -Systolic pressure: Highest during a cardiac cycle. Measured after heart contracts (systole) and blood is ejected into arterial system. -Diastolic pressure: Lowest. Measured when heart is relaxed and blood is returning to heart. -Pulse pressure: The difference between systolic and diastolic pressure. Most important determinant is stroke volume. As blood is ejected from the left ventricle into the arterial system, systolic pressure increases because of the relatively low capacitance of the arteries. Because diastolic pressure remains unchanged during ventricular systole, the pulse pressure increases to the same extent as the systolic pressure. -Decreases in capacitance, such those that occur with the aging process, cause increases in pulse pressure -Mean arterial pressure: Average arterial pressure with no respect to time. Can be calculated approximately as diastolic pressure plus one third of pulse pressure.

When the Bundle of His reaches the myocardial tissue, what is it called?

Purkinje fibers

What part of the stomach is highlighted?

Pyloric part

What part of the stomach is this?

Pyloric sphincter

Duodenal ulcers can be surgically treated through vagotomy following ______ or _______ or the specific vagotomy of the ________ ________.

Pyloroplasty, gastrojejunostomy, parietal cells

What do the cortex and medulla contain?

Pyramids

R = 8nL/pi.r^4

R= resistance n= viscosity of blood l=length of blood vessel r4= radius of blood vessel to the fourth power

Hematopoiesis

RBC formaiton

When can the shape of blood cells change?

Radiation therapy when someone has cancer

Ventricular tachycardia

Rapid sequence of premature ectopic ventricular impulses. Beginning with ES. Ventricular filling and cardiac output decrease and ventricular fibrillation ensues. High frequency, uncoordinated twitching. Unless treated, failure to eject blood can be just as dangerous as cardiac arrest (ventricular fibrillation). Multiple QRS complexes without visible P or T waves. QRS will appear wider than normal.

(CP) Capacity depends on

Rate of elastic fiber that normally exists in the wall of blood vessels

Distal tubule/Collecting duct

Reabsorb 8% of filtered sodium. Early: reabsorbs salt by sodium + chloride cotransporter. Impermeable to water. Cortical diluting segment. Late: Two cell types -Principal cells: Reabsorb sodium and water. Secrete potassium. Aldosterone increases sodium reabsorption and increases potassium secretion. Depends on factors such as dietary K+, aldosterone levels, acid-base status, and urine flow rate Antidiuertic hormone increases water permeability by directing secretion in water channels in luminal membrane. In absence of ADH, virtually impermeable to water. -Alpha intercalated cells: Secrete hydrogen by hydrogen ATPase (stimulated by aldosterone). Reabsorb potassium by a hydrogen-potassium-ATPase.

Magnesium (renal regulation)

Reabsorbed in proximal tubule, thick ascending loop of Henle, and distal tubule. In thick ascending limb: magnesium and calcium compete (hypercalcemia causes an increase in magnesium secretion, inhibiting absorption)

Thick ascending loop of Henle

Reabsorbs 25% filtered sodium. Contains Na+-K+-2Cl- cotransporter in the luminal membrane. Impermeable to water. Salt is reabsorbed without water. Resulting in tubular fluid sodium and osmolarity decrease to less than their concentrations in plasma. Called diluting segment.

Early proximal tubule features

Reabsorbs sodium and water with HCO3, glucose, amino acids, phosphate, and lactate. Sodium is reabsorbed by cotransport with glucose, AAs, phosphate, lactate. Cotransport accounts for reabsorption of all filtered glucose and AAs. Sodium can also be reabsorbed by countertransport via sodium-hydrogen exchange. Directly linked to the reabsorption of filtered HCO3

Renal Acid-Base Regulation

Reabsorption of filtered HCO3: Occurs primarily in proximal tubule. Produced from CO2 and H2O. Form H2CO3 carbonic acid, catalyzed by intracellular carbonic anhydrase. H2CO3 will dissociate into HCO3 and H+. H+ is secreted into the lumen via the sodium hydrogen exchange mechanism in luminal membrane. HCO3 is reabsorbed. In the lumen: secreted H+ combines with filtered HCO3 to form H2CO3. Dissociates into CO2 and H2O, catalyzed by brush border carbonic anhydrase. CO2 and H2O diffuse, resulting in reabsorption. Does not result in secretion of H+

Erythrocytes

Red blood cells. Contain hemoglobin. Carries 4 oxygen molecules, iron. Important for oxygen/CO2 transportation. Erythropoiesis: formation of erythrocytes. Leukopoiesis: formation of leukocytes. Conversion of reticulocyte into erythrocytes requires 3 factors: vitamin B-12, iron (Fe2+) and folic acid. *Deficiency of folic acid in pregnancy can affect the development of CNS (brain) and can cause encephalopathy (brain cannot develop). Deficiency in any of these can lead to anemia.

Phase 3

Relaxation phase (opening of K+ channels and it leaves the cell)

What do B lymphocytes do?

Release antibodies

When is erythropoietin released?

Released by kidneys when RBCs are low due to decreased O2

Treatment for Conns diseases

Remove tumor and treat hypertension

Division of renal artery

Renal artery becomes interlobar artery → arcuate artery → cortical artery → radial artery → afferent arterioles, which then enters into Bowman's capsule → glomerular capillary → efferent arteriole → peritubular capillary → renal vein Artery → capillary → vein

Renal Blood flow

Renal artery-> segmental arteries-> interlobar arteries-> arcuate arteries -> interlobular arteries-> afferent arterial ->glomerulus -> efferent arterial -> peritubular cappilaries (PCT and DCT) -> vasa recta (loop of henley) -> Interlobular veins -> arcuate veins -> interlobar veins -> segmental veins -> renal vein.

What is the treatment for jaundice?

Replace folic acid, iron, and give antibiotic for bacterial infection

Phase 3 (ventricles, atria, purkinje)

Repolarization. Calcium conductance decreases and potassium conductance increases and therefore predominates. The high potassium conductance results in a large outward potassium current which hyperpolarizes the membrane back toward the potassium equilibrium potential.

Phase 3 (SA node)

Repolarization. Caused by an increase in potassium conductance. Results in outward potassium current causing repolarization.

What problems does hypoxemia lead to?

Respiratory problems, pain in joints and abdominal cavity, and can affect other blood stem cell production in bone marrow

Phase 4 (ventricles, atria, purkinje)

Resting membrane potential. Period during which inward and outward currents are equal and the membrane potential approaches the potassium

Arrhythmia

Results from abnormalities in impulse formation or in impulse conduction. Disturbances in the formation of impulses lead to change in the sinus rhythm

During vomiting, if the upper esophageal sphincter remains closed, ______ occurs.

Retching

Lymph

Returns any filtered protein to the circulation. Undirectional flow: one-way flap valves permit interstitial to enter but not leave. Flow through larger vessels is aided by one-way valves and skeletal muscle contraction. -Edema: Occurs when volume of interstitial fluid exceeds the capacity of lymphatic to return it to circulation. Caused by excess filtration or blocked lymphatics.

Atrial tachycardia

Rhythm disturbance. Arises in the atria. Heart rates are highly variable with a range of 100-250 bpm. Usually regular.

Tricuspid valve

Right atrioventricular valve. Separates the right atrium and right ventricle. Regulates blood flow.

Where is the AV node located?

Right atrium on interatrial septum

Signs and Symptoms of Leukemia...

Risk for infection, inflammation, fever, weakness Joint pain and abdominal pain (Because of infiltration of antibodies in joints)

Secretin is secreted by the ________ cells of the ________.

S, duodenum

In response to acidic chyme, the __ cells of the duodenum secrete _____.

S, secretin

Action potential in ______, and ______, have similar steps (3 steps)

SA and AV node

The vagus nerve is the primary parasympathetic nerve of the stomach while the _____ nerves are the primary sympathetic nerves of the stomach.

Splanchnic

Overproduction of RBCs lead to enlargement of which organs?

Spleen and liver

By strong contraction of ventricle, there is ejection of what?

Stoke volume into aorta which is about 70 mL

The frequency of slow waves is lowest in the _______ ( ____/minute) and highest in the _______ ( ____/minute).

Stomach, 3, duodenum, 12

Blood plasma

Straw colored, sticky fluid portion of blood. ~90% water. Contains: sodium ions, nutrients, hormones, enzymes, antibodies, wastes, and proteins. Three main proteins: ablumin, globulin, fibrinogen. Albumin: Osmotic concentration. Produced by liver 60-80%. Globulin: Alpha/Beta: produced by liver, transports lipids and fat-soluble vitamins. Gamma: immune function. Fibrinogen: Blood clotting.

Blood volume is what other kind of volume?

Stress volume

The pharynx, the upper third of the esophagus and the anal sphincters are composed of _______ muscle.

Striated

Papillary muscle

Structure from which chordae tendineae originate. 3 in right ventricle and 2 on left ventricle.

Defects in which plexus lead to malabsorption?

Submucosal

The ________ plexus primarily controls secretion and bloodflow.

Submucosal

The ________ and __________ comprise the enteric nervous sytem of the GI tract

Submucosal plexus, myenteric plexus

Preganglionic parasympathetic fibers synapse in the ________ and ________ plexuses.

Submucosal, myenteric

Ventrciular extrasystole (ES)/Infranodal extrasystole

Ventricular premature complexes (VPC) are ectopic pulses originating from area distal to the His Purkinje system. VPC are most common in ventricular arrhythmia. The ES is deformed in the QRS complex. Two common mechanisms: Automaticity, and reentry. Frequent VPC become a form of ventricular tachycardia (rapid heartbeat) -Automaticity: Development of a new site of depolarization in non-nodal ventricular tissue, which can lead to VPC. Increased automaticity could be due to electrolyte abnormalities or ischemic (restriction in blood supply) myocardium -Reentry circuit: Typically occurs when slow conducting tissue (ex: infarcted myocardium) is present adjacent to normal tissue. Could be due to damaged myocardium, as in the case of a healed MI. Causes: Ischemia, digoxin (medication that increases heart contraction), myocarditis, cardiomyopathy hypertrophic (or dilated), hypoxia, hypercapnia (CO2 poisoning), mitral valve prolapse, smoking, alcohol, cocaine use, caffeine, magnesium and potassium deficiency, calcium excess, thyroid problems, heart attack. -Symptoms: Chest pain, faint feeling, fatigue, hyperventilation (after exercise).

T wave

Ventricular repolarization. Longer than depolarization.

What does the esophagus run along?

Vertebral column

Venous pressure

Very low. Veins have high capacitance and therefore, can hold large volume of blood at low pressure.

Pernicious anemia

Vitamin B12 deficiency

What three factors are needed to convert reticulocytes into Erythrocytes?

Vitamin B12, folic acid, and iron (Fe2+)

Stroke volume

Volume ejected from the ventricle on each beat. Stroke volume= end-diastolic volume - end-systolic volume

What is preload?

Volume of blood. which exists in ventricle at the end of relaxation phase of ventricle

A wave of reverse peristalsis begins in the small intestine and moves content in the orad direction during _______.

Vomiting

leukocytes =

WBCs

Where are chemoreceptors located for this cycle?

Walls of large blood vessels

What is the Henle loop impermeable by?

Water

Do you know the cause of secondary hypertension?

We know the cause and can easily treat it

Sinusoids

Weak, leaky capillaries found in liver and spleen.

Increased immature WBCs will lead to what risks?

Weakened immune system, risk of infection, and inflammation

Prolonged hypoxemia leads to what problems in a patient?

Weakness, weak immune system, risk for infection, and increased heart rate/tachycardia/palpitations

When is Renin-angiotensin-aldosterone system activated?

When blood pressure and blood Na+ level is low, it leads to secretion of renin from kidney

When is a baroreceptor stimulated?

When blood pressure is low

What is mitral stenosis?

When mitral valve cannot open fully properly

Effective refractory period (ERP)

When there is second stimulus during phase 1 or 2, that second stimulus has effect but the cell cannot show reaction to it

Relative refractory period (RRP)

When there is second stimulus during phase 3 or 4, the cell membrane accepts it and shows reaction to that stimulus

Absolute refractory period (ARP)

When there is second stimulus to cell membrane during depolarization phase, the cell cannot accept that stimulus and rejects the reaction

Cardiac output

When ventricle contracts and ejects large amount of blood into large blood vessel per minute, which is approximately about 5 L

21. The answer is D. First, the acid-base disorder must be diagnosed. Alkaline pH, with increased HCO3- and increased PCO2, is consistent with metabolic alkalosis with respiratory compensation. The low blood pressure and decreased turgor suggest extracellular fluid (ECF) volume contraction. The reduced [H+] in blood will cause intracellular H+ to leave cells in exchange for extracellular K+. The appropriate respiratory compensation is hypoventilation, which is responsible for the elevated PCO2. H+ excretion in urine will be decreased, so less titratable acid will be excreted. K+ secretion by the distal tubules will be increased because aldosterone levels will be increased secondary to ECF volume contraction. Which of the following responses would also be expected to occur in this patient?(A) Hyperventilation(B) Decreased K+ secretion by the distal tubules(C) Increased ratio of H2PO4- to HPO4-2 in urine(D) Exchange of intracellular H+ for extracellular K+

Which of the following responses would also be expected to occur in this patient?(A) Hyperventilation(B) Decreased K+ secretion by the distal tubules(C) Increased ratio of H2PO4- to HPO4-2 in urine(D) Exchange of intracellular H+ for extracellular K+

a 60 year old businessman is evaluated by his physician who determines that his bp is significantly elevated at 185/130, lab tests reveal an increase in plasma renin activity, plasma aldosterone levels, and left renal vein renin levels, his right renal vein renin level is decreased, what is the most liekly cause of the pt's HTN?

left renal artery stenosis

in which of the following situations is pulmonary blood flow greater than aortic blood flow?

left to right ventricular shunt

V3-V6 detects

left ventricle

strongest chamber of the heart

left ventricle - last part of heart to pump oxygenated blood driving forces of body circulation

Why does the patients heart fail according to sterlings law

left ventricle cannot receive sufficient blood so according to Starling's law we need end-diastolic volume, but we cannot build it up

Hormones

little chemical messengers

during exercise tatal peripheral resistance (TPR) decreases because of the effect of

local metabolites on skeletal muscle aterioles

20% of potassium is absorbed by

loop of henle

Thrombocytopenia

low platelet count(clotting)

Veins

low pressure thin walled have highest portion of blood in body "unstressed volume" Alpha 1 adrenergic receptors

Stress can cause

many types of disorders and autoimmune diseases

venules

merged capillaries

Thyroid hormone also controls

metabolism and temperature, stimulates alpha-1 and beta-1 adrenergic receptors

microcytic vs macrocytic anemia

micro = iron deficiency Marco = vit B12 or Folate deficiency

At the end of the two pyramids there is

minor calyx (which together form the major calyx)

Where does the collecting tubule open into?

minor calyx, major calyx, and then ureter

What receptors in the parasympathetic cause the negative effect?

muscarinic receptors

Ca+2 is important for

muscle contraction

Cortisol breaks down protein which causes

muscles (which is full of protein) in lower and upper limbs to thin

skeletal muscle pump

muscles press against thin-walled veins to pump blood

distribuiting arteries are

muscular arteries

all granulocytes originate from

myeloblasts

Erythroblast production intermediates in order

myeloid stem cell proerythroblasts normoblast reticulocyte erythroblast

Having 140 mL of blood on your myocardial wall put tension on

myocardial wall which increases the length of the muscle fiber

Inside the medulla and cortex, there are millions of what?

nephrons

Atherosclerosis inactivate/inhibits the natural vasodilator such as

nitric acid and attracts WBC which cause inflammation

Resistance depends on what?

on TPR, intersectional area, diameter of blood vessel, length of blood vessel, and viscosity (when blood is concentrated, it shows more resistance to blood flow)

After depolarization of the cell, it leads to

opening of Ca+2 channels and Ca+2 goes into the cell

After severe dehydration there is a stimulation of what kind of receptors?

osmoreceptors

ADH has what kind of receptors? Blood pressure has what kind of receptors?

osmoreceptors baroreceptors

Over-secretion of noradrenaline leads to

overactivation of alpha-1 → vasoconstriction

What is Cushings syndrome?

overproduction and over-secretion of adrenal cortex hormones

Low __ in the stomach acts as negative feedback in gastric H+ secretion.

pH

Parathyroid Glands

parathyroid cells secrete parathyroid hormone increases calcium levels ~stimulate osteoblasts, breaking down bone realeasing calcium ~KIdneys inhance calcium reobsorbtion ~intestine, promotes activation of vitamine D which helps us absorb calcium.

What inhibits phosphate absorption?

parathyroid hormone

the tendency for blood flow to be turbulent is increased by

partial occlusion of a blood vessel

What is hematocrit?

percent of blood volume that is RBCs

After filtration of 180 L the

peritubular capillary reabsorbs fluid and ions

during which phase of the ventricular action potential is the conductance to Ca2+ highest?

phase 2

during which phase of the ventricular action potential is the membrane potential closest to the K+ equilibrium potential?

phase 4

which phase of the ventricular action potential coincides with diastole?

phase 4

Blood = _____ + _____

plasma + formed elements

What do V2 receptors activate?

pores that help with absorption of urine into capillaries

Why is pressure that exists in arterial system much higher than the venous system?

pressure that exists in aorta is due to strong contraction of left ventricle

What is the function of nephron?

production of urine

Myeloid cycle?

proerythroblast → early erythroblast → late erythroblast → normoblast → reticulocyte

Prolactin (PRL)

promotes lactation

67% of potassium is absorbed by

proximal tubule capillaries

What do nephrons consist of?

proximal tubule, loop of Henle (U-shaped), distal tubule, and collecting tubule

Luekemia cells grow

rapidly and do not stop when they should

Myeloid tissues are

red bone marrow tissue in long bones (sternum, pelvis, bodies of vertebrae _produce all other blood cells except lymphoid

during which phase of the cardiac cycle is aortic pressure highest?

reduced ventricular ejection

Growth hormone (GH)

regulates metabolism and body growth

K+ is important for

relaxation phase/repolarization phase

Between the two pyramids there is the

renal column

The major calyx join together to form

renal pelvis

Low blood pressure and low blood sodium stimulates

renin angiotensin aldosterone system and aldosterone increases absorption of Na+, Cl-, and H2O by capillary and increases blood pressure

Pulmonary system

reoxygenates blood Carries CO2 blood away from heart, to lungs takes fresh O2 blood from lungs towards heart

T waves represent

repolarization of ventricles (relaxation)

TPR ((total peripheral resistance)

resistance that exists in capillary

Blood transports what?

respiratory gasses, metabolites, nutrients

Build up of blood in left atrium causes

retroflow of blood into pulmonary veins

What are the kidneys covered by?

retroperitoneal membrane

What 5 components is the urinary system composed of?

right (lower bc of liver) and left kidney, Ureter, Urinary Bladder, Urthera.

Where can you hear the aortic valve?

right 2nd intercostal space

Where is the AV node located

right atrium between two atrial septum

The fragile blood vessels when broke will cause

rupture and local bleeding

primary

secrete endocrine hormones as there primary function (pituitary gland, thyroid gland, adrenal gland)

Adrenocorticotropic hormone (ACTH)

secretes androgen mineralocoroticods, androgens.

BBB is a

selectively permeable capillary O2, CO2, and some anesthetics can pass

V3 and V4 detects

septum (interventricular membrane) and lateral wall of left ventricle

Fluid from clotted blood is?

serum

blood flow to which organ is controlled primarily by the sympathetic nervous system rather than by local metabolites?

skin

Capillary has resistance to blood flow because of what?

small diameter and intersectional area, which shows resistance to blood flow and participates or builds up blood pressure in aorta

Capilaries are

smallest BVs RBCs pass 1 ata time

Capillary lacks what muscle?

smooth muscle

The wall of artery and vein contain

smooth muscle which controls ANS and blood pressure

Where are RBC's degraded?

spleen

All blood cells originate from

stem cells in the bone marrow

Aldosterone binds to its specific receptor which leads to

stimulation of capillary

Hemostasis

stoppage of bleeding fibrinogen is important

Arterial system is under pressure so it is called

stress volume

When blood pressure is high there is stimulation of what kind of receptors?

stretch receptors

After the 2 factors effect Ca2+ there is a

strong myocardial contraction

What two nuclei release ADH?

supraoptic and paraventricular

Noradrenaline stimulates alpha-1 and beta-1 receptors, for this reason patient with pheochromocytoma has

sweating, palpitations, and heart disease

Change in partial pressure of either oxygen or carbon dioxide will lead to the stimulation of

sympathetic system

Thyroid hormones control

the CNS, bone development, and thyroid development in fetus

if blood glucose is higher than 200mg/dl but lower than 350mg/dl?

There is some reabsorption of glucose and excretion of glucose

Why don't triglycerides stimulate the release of CCK?

They cannot cross the intestinal membrane

What do antibodies do?

They detect antigents and forms antigen-antibody complex then other WBCs (white blood cells) can destroy the antigen

What should the lungs do in hypoxemia?

They should compensate and cause hyperventilation

Myocardium

Thick middle muscle layer of the heart. Muscle underneath fat tissue

For this reason it has

Thicker walls than venous system

Chordae tendineae

Thin bands of fibrous tissue that attach to the valves in the heart and prevent them from inverting. Connect AV valves to papillary muslces.

Why go for diuretic in the case of renal disease?

To eliminate urine from the body which is helpful for hypertension (helps accelerate renal filtration rate)

The contractions that occur as a result of slow waves are called basal or ________ contractions.

Tonic

____________ contractions occur in the lower esophageal sphincter, orad stomach, and ileocecal and internal anal sphincters.

Tonic

Body fluids

Total body water (TBW): ~60% body weight. Highest in newborns and adult males, lowest in adult females and in adults with large amount of adipose tissue. Distribution: Plasma: 1/4 (25%) of ECF. Interstitial fluid: 3/4 (75%) of ECF 60-40-20 rule: TBW: 60% body weight ICF: 40% body weight ECF: 20% body weight

When there is a hole between the trachea and the esophagus. In newborns can cause severe respiratory distress as milk enters respiratory tract.

Tracheoesophageal fistula

What are the 3 main functions of the circulatory system(blood)?

Transportation, regulation, protection

Circulatory system function

Transportation: respiratory gases, nutrients, metabolites. Regulation: hormonal, temperature Protection: Clotting: protects against blood loss from injury and foreign microbes. Immune function. Blood volume: Male: 5-6 liters Female: 4-5 liters

What does beta globulin do?

Transports proteins

Treatment for glomerulonephritis

Treatment for edema: Use diuretic (accelerates urine production) In the case of bacterial infection, treat with antibiotics Treat pain with painkillers Treat hypertension with anti-hypertensive drugs

T/F: Mucoid cells produce mucous and some pepsinogen

True

T/F: Slow waves occur spontaneously

True

T/F: Spike potentials are action potentials

True

T/F: Stretching of GI tract membranes can cause depolarization.

True

T/F: Vagotomy is capable of blocking ALL H+ secretion

True

Blood vessels

Tubelike structures that carry blood throughout the body. Composed of three layers: Tunica intima: composed of simple squamous epithelium, Tunica media: sheets of smooth muscle, causing vasoconstriction (contraction) and vasodilation (relaxation), Tunica externa: Composed of connective tissue. Lumen: Central blood-filled space of a vessel. Types: Arteries: carries blood away from heart, veins: carry blood to heart, capillaries: smallest blood vessels and site of exchange of molecules

Pheochromocytoma

Tumor in the adrenal medulla

Pheochromocytoma

Tumor of the adrenal medulla. Symptoms: Excess secretion of adrenaline and noradrenaline, hypertension, headache, vomiting, nausea, sleep disorder, sweating, muscle spasms, palpitations, tachycardia , heart disease.

Cushings syndrome

Tumor, infection or auto-immune disorder in the adrenal cortex results in excess secretion of aldosterone, androgen, and cortisol. Symptoms: Increase sodium in blood, decreased potassium in blood, increased blood pressure, headache, vomiting, nausea, sleep disorder, nose bleeds, anxiety. If increased cortisol: increased diastolic/systolic pressure, TPR, and increased blood glucose (hyperglycemia), obesity/weight gain. If increase in androgen: Abnormal facial hair growth in females, decrease in estrogen and progesterone: irregular menstrual cycle and ovulation, possibility infertility. Treatment: Surgical removal of tumor. Antibiotics for infection. Medication to control symptoms.

Pituitary gland

Two major lobes -anterior (adenohypophysis) ~makes its own hormones -posterior (neurohypophysis) ~hypothalamus makes and synthesizes its hormones ~storage site for hormones

Second degree AV node block duration is

Two successful P waves and then QRS

Leukocytes

Two types: Granular: basophil, eosinophil, neutrophil. Agranular: lymphocytes, monocytes.

Lymphocytes

Two types: T and B cells. White blood cells. 20-45%. Nucleus stains dark purple. Most important cells in the immune system T cells: attack foreign cells directly B cells: multiply to become plasma cells and secret antibody.

Where can you put nitroglycerin to make it work faster?

Under the tongue for faster absorption by lingual nerve

Where does the accumulation of cholesterol, lipid and Ca2+ accumulate?

Underneath the endothelial layer of blood vessels walls

Contractile tissue of the GI tract is ______ smooth muscle.

Unitary

Phase 0 (SA node)

Upstroke of action potential. Caused by an increase in calcium conductance. The increase causes an inward calcium current that drives the membrane potential toward the calcium equilibrium potential.

Water dilutes

Urine

What does the ADH bind to?

V2 receptors

Neurocrine that produces relaxation of the GI smooth and has the same effects as secretin.

VIP

What are the three nuerocrines found in the GI tract?

VIP, GRP, enkephalins

The relaxation of the LES is mediated by the neurocrine ______ through the _____ nerve.

VIP, vagus

_______ stimulation increases the strength(?) of gastric contraction while ______ stimulation decreases it.

Vagal/parasympathetic, sympathetic

Receptive relaxation is absent is a ______ is undergone.

Vagotomy

______ is used for treating gastric ulcers that are unresponsive to drugs.

Vagotomy

Reflexes in which both a/efferent pathways are contained in the vagus nerve are called what?

Vagovagal reflexes

The right and left ______ nerves accompany the esophagus.

Vagus

This nerve innervates the esophagus, stomach, pancreas, and upper large intestine.

Vagus

Parasympathetic impulses are carried by which two nerves?

Vagus, Pelvic

Nervous secretion of gastric juice is mediated by the ______ nerve which is activated by _____, ______, and ______.

Vagus, taste, smell, sight

Block AgII using

Valsartan

Heart murmurs can be attributed to what problem?

Valve problems such as valve stenosis or valve insufficiency

What are Alpha-1 receptor functions?

Vasoconstriction

Alpha-1 stimulation leads to

Vasoconstriction and thus the increase in blood pressure

The wall of artery has more elastic fibers than

Veins

2nd part of Frank-Sterling relationship

Venous return release large amount of blood into atrium, which is about 70 mL

What happens during isometric contraction?

Ventricle is ready to have strong contraction, which moves pressure fro 0-100 mmHg

What is ready for ejection and what is ejected during ventricular ejection?

Ventricles and the stoke volume is ejected (70 mL)

What is the second step of the cardiac cycle?

Ventricular ejection (2-3)

What is the last step of the cardiac cycle?

Ventricular filling (4-1)

When it comes to artery/arteriole, it drops to what mmHg?

50

Where does most of the absorption in the urea occur?

50% in proximal tubule capillaries

Urea (renal regulation)

50% reabsorbed passively through proximal tubule. Rest is impermeable. ADH increases permeability of inner medullary collecting ducts

How many bmp is bradycardia

50-60 bpm

Normal heart beat

50-99 times per minute at rest.

Where can you hear the mitral valve?

5th intercostal space , left midclavicular line

6. The answer is D. Decreases in arterial PCO2 cause a decrease in the reabsorption of filtered HCO3- by diminishing the supply of H+ in the cell for secretion into the lumen. Reabsorption of filtered HCO3- is nearly 100% of the filtered load and requires carbonic anhydrase in the brush border to convert filtered HCO3- to CO2 to proceed normally. This process causes little acidification of the urine and is not linked to net excretion of H+ as titratable acid or NH4+.

6. The reabsorption of filtered HCO3-(A) results in reabsorption of less than 50% of the filtered load when the plasma concentrationof HCO3- is 24 mEq/L(B) acidifies tubular fluid to a pH of 4.4(C) is directly linked to excretion of H+ as NH4+(D) is inhibited by decreases in arterial PCO2(E) can proceed normally in the presence of a renal carbonic anhydrase inhibitor

the following measurments were obtained in a male patient: HR 70 pulmonary vein O2= .24mL O2/mL pulmonary artery O2= .16 mL O2/mL whole body O2 consumption = 500mL What is this patient's cardiac output?

6.25 L/min

Calcium (renal regulation)

60% of plasma calcium is filtered across glomerulus capillaries. Proximal tubule and thick ascending limb reabsorb more than 90% of filtered calcium by a passive process coupled with sodium reabsorption. Distal tubule and collecting duct reabsorb 8% by an active process. *PTH increases calcium by activating adenylate cyclase in distal tubule

What is the percentage of Na+ is absorbed by proximal tubule capillary?

67%

The salivary glands are innervated by cranial nerves ___ and ___.

7, 9

Systolic volume

70 mL

The ventricle, before the contraction of the atrium, has how many mL of blood in it?

70 mL of blood

By contraction of atrium, how much mL of blood get into where?

70 mL of blood into the ventricle

End-systolic volume

70mL

In pregnancy the ___ month is where the bone marrow becomes hematopoietic and main source of blood.

7th

8. The answer is A. Total daily production of fixed H+ from catabolism of proteins and phospholipids (plus any additional fixed H+ that is ingested) must be matched by the sum of excretion of H+ as titratable acid plus NH4+ to maintain acid-base balance.

8. To maintain normal H+ balance, total daily excretion of H+ should equal the daily(A) fixed acid production plus fixed acid ingestion(B) HCO3- excretion(C) HCO3- filtered load(D) titratable acid excretion(E) filtered load of H+

Where does more the absorption in phosphate occur?

85% in proximal tubule

Caused when the LES does not relax and food builds up in the esophagus.

Achalasia

Dysphagia (difficulty swallowing) and retrosternal pain are symptoms of _______.

Achalasia

Parts of the large intestine:

Cecum, ascending colon, transverse colon, descending colon, sigmoid, rectum

This phase of gastric secretion results in 20% of the acid secretion associated with a meal.

Cephalic

To stimulate the activation of partial pressure of oxygen or carbon dioxide, we need to stimulate what receptor?

Chemoreceptors

ECG lead: Views

Chest electrode placement: -V1: Fourth intercostal space to the right of the sternum -V2: Fourth intercostal space to the left of the sternum -V3: Between leads V2 and V4 -V4: Fifth intercostal space at midclavicular line. -V5: Level with V4 at left anterior axillary line -V6: Level with V5 at left midaxillary line (directly under the midpoint of the armpit) Chest Leads: a: V1 and V2- right ventricle b: V3 and V4- septum/ lateral left ventricle c: V5 and V6- anterior/ lateral left ventricle

Symptoms of arrhythmia

Chest pains and hypoxemia (Low oxygen levels in blood)

Gland in the body and fundus of the stomach are elongated and include which three types of cells?

Chief, parietal, mucoid

Primary bile acids are synthesized from ______ by hepatocytes.

Cholesterol

Zollinger-Ellison syndrome is treated using which two drugs?

Cimetidine (H2 receptor blocker), and Omeprazole (H+ pump inhibitor)

A patient was recently treated with Omeprazole, but you've discovered that H+ levels are far too low for effective digestion (hypochlorhydria). Your next option for treatment would be ________.

Cimetidine/Atropine

Contraction causes a decrease of the diameter of the lumen of the GI tract.

Circular muscle

A patient is exhibiting portal hypertension and, as a result, splenomegaly. Accumulation of fluid in the abdomen (ascites) is also observed. What is the correct diagnosis?

Cirrhosis

Treatment for kidney stones?

Citrate deficiency - go for citrate replacement Drinking water for spontaneous elimination of stone Ca2+ channel blockers, such as Verapamil, to control Ca2+ channels Nonsteroidal anti-inflammatory drugs, such as aspirin, for pain

Where does the pressure build up during isovolumetric relaxation?

In the atria, which is higher than ventricular pressure

Where are baroreceptors located?

In the wall of blood vessels

Where are the osmoreceptors located in this cycle?

In the wall of large blood vessels

Build up pressure and blood supply will lead to what for oxygen and carbon dioxide levels?

Increase blood oxygen molecules and decrease carbon dioxide levels

Frank Starling Relationship/Law

Increase in stroke volume and cardiac output that occur in response to an increase in venous return or end-diastolic volume. Based on the length tension relationship in the ventricle. Increases in end-diastolic volume will cause an increase in ventricular fiber length, which produces an increase in developed tension. The mechanism that matches cardiac output to venous return. The greater the venous return, the greater the cardiac output. -Changes in contractility shift the Frank-Starling curve upward (increased contractility) or downward (decreased contractility). -Increases in contractility cause an increase in cardiac output for any level of right atrial pressure or end-diastolic volume. -Decreases in contractility cause a decrease in cardiac output for any level of right atrial pressure or end-diastolic volume.

High levels of cortisol lead to

Increase is TPR and systolic/diastolic pressures which in turn leads to hypertension

Cardiac glycoides

Increase the force of contraction by inhibiting sodium-potassium-ATPase in the myocardial cell membrane. Resulting in an increase in intracellular sodium, diminishing the sodium gradient across the cell membrane. Sodium-calcium exchange depends on the size of the sodium gradient, producing an increase in intracellular calcium.

Signs of Pheochromocytoma

Increased adrenaline and noradrenaline Hypertension Headache Vomiting Nose bleeding Vision problems Sweating, palpitations, and heart disease

Increased aldosterone leads to

Increased blood Na+, Decreased K+, and hypertension

Length-tension relationship by sterlings law

Increased greater venous return leads to increased cardiac output

When a person moves from a supine position to a standing position, which of the following compensatory changes occurs?

Increased heart rate, and increased contractility

Signs/Symptoms of coarctation

Increased pressure in head and neck Low blood pressure in lower limb

Thyroid and Parathyroid conditions

Increased secretions of T3 and T4 can result in increased sensitivity of sympathetic receptors alpha-1 and beta-1 norepineprhine. Overstimulation of these receptors, especially alpha-1, causes hypertension. Symptoms: severe headaches, vomiting, nausea, sleep disorders, nose bleeds, problems with vision, anxiety, vertigo, weakness, fatigue, heart palpitations, increased heart rate and contractility in the myocardium.

Due to the increased length of muscle fiber created from the tension of the 140 mL of blood there are 2 factors. What is the first one?

Increases Ca2+ release from SR into cytoplasm

What are the 2 factors from the increased length of myocardial fiber?

Increases Ca2+ released from sarcoplasmic reticulum and Increases Ca2+ sensitivity to troponin C

Irritation or disorder of bone does what to the body?

Increases RBCs, WBCs, platelets and hematocrit

Sympathetic effects on heart

Increases heart rate, conduction velocity, and contractility (beta-1 receptors). Alpha-1 receptor for constriction of splanchic, skin, and skeletal muscle. Beta-2 receptor for relaxation of skeletal muscle. Norepinephrine is the neurotransmitter, acting as Beta-1 receptors. -Positive chronotropic effect: Increases heart rate by increasing the rate of phase 4 depolarization. More action potentials occur because threshold potential is reached more quickly. Mechanism is the inward sodium current. -Positive dromotropic effect: Increases conduction velocity through AV node. Action potentials are conducted more rapidly from atria to ventricles and ventricular filling may be compromised. Decreases the PR interval. Mechanism is increased inward calcium current. -Positive intropism: Increases the inward calcium current during the plateau of each cardiac action potential. It increases the activity of the calcium pump of the SR as a result more calcium is accumulated by the SR and thus more calcium is available for release in subsequent beats.

Aldosterone

Increases potassium secretion. Mechanism is increased sodium entry into cells across luminal membrane and increased pumping of sodium out of cells by sodium potassium pump. Stimulation of the pump will increase potassium increase uptake into principal cells. Increase intracellular potassium concentration and driving force. Also increases the number of luminal membrane potassium channels. Hyperaldosteronism: Increases potassium secretion causing hypokalemia Hypoaldosteronism: Decreases potassium secretion and causes hyperkalemia

Smooth muscle is under control by what system and receptors?

autonomic nervous system (ANS) especially by alpha-1 and beta-2 adrenergic receptors

arteries

away from heart

Lymphocytic

based on lymphoblastic

Why does the pressure drop in the capillaries level?

because of smaller diameter and intersectional area

Why does and elevated or depressed ST segment show a myocardial infarction?

because there is something wrong with completion of ventricular contraction or something wrong between ventricular contraction and relaxation

Valves that surround the Left atrium (carrying Oxygenated blood)

bicuspid?mitral, LV, aortic semilunar

What is the role of iron in the heme?

binding to oxygen molecules (4/heme) and bringin oxygen from lungs to body, bringin CO2 waste back out in breath

Overproduction of RBCs lead to what?

blood becomes concentrated and accumulate in blood vessels → increases blood pressure (hypertension)

Stressed volume is

blood contained in arteries

Q = P/R

blood flow equals pressure resistance divided by total gradient divided by resistance

What are these two hormones involved in?

blood pressure, sweat glands, and heart function

Cardiac output is the amount of

blood, which is ejected into aorta per min (= about 5L)

carbon dioxide regulates blood flow to which one of the following organs?

brain

Somatostatin and prostoglandins inhibit the secretion of H+ by blocking the production/accumulation of ______.

cAMP

The second messenger for histamine is ______.

cAMP

The second messenger in sympathetic stimulation of saliva production is _______.

cAMP

Deficiency of these vasodilators

can cause hypertension

Luekemia

cancer of WBCs aka "lymphoblastic" or "myeloblastic" condition arises in bone marrow instead of making leukocytes makes luekemia cells

Arteriole is bigger than

capillaries

Gas exchange only occurs in what?

capillaries

What is nephron surrounded by?

capillaries

Nephron is surrounded by

capillary

Why does mmHg drop during the capillary level?

capillary lacks smooth muscle, has more intersectional area, and a smaller diameter

CO = (MAP - RAP) / TPR

cardiac output equals mean arterial pressure minus right arterial pressure divided by Total Peripheral Resistance

the formula of blood pressure

cardiac output x TPR (total peripheral resistance is the resistance that exists in capillary)

ANS controls

cardiovascular system

Hypercalcemia

cause hypertension because Ca2+ is important ion for smooth, skeletal, and cardiac muscle contraction

When you check for a bacterial infection in blood, what do you look for?

check erythrocyte sedimentation rate, and transesophageal echocardiogram

How to treat luekemia?

chemotherapy - kill bad cell + strong anti-cancer drugs interferon-alpha therapy-slow prodcution of bad cells, promote anti-luekemia activity stem cell transplantation- high doses of chemotherapy and radiation therapty radiation therapy- kill cancer cells

Final urine is from

collecting tuble

What is mitral valve prolapse?

congenital problem of mitral valve

What can cause mitral stenosis?

congenital, tumor (it can close mitral valve), infection (bacterial, viral, fungal), or autoimmune

Blood is classified as ___ tissue

connective

Vasoconstriction

constriction of blood vessels inc BP

Excretion from renal systems means

contents is excreted into urine

QT interval means

contraction and relaxation of both ventricles

Corticosteroid is the same as

cortisol

When we have stress we have high levels of

cortisol

a person's ECG show two p waves preceding each QRS complex, the interpretation of this pattern is

decreased conduction through the AV node

an acute decrease in arterial blood pressure elicits which of the following compensatory changes?

decreased firing rate of the carotid sinus nerve

In an ECG on a person shows ventricular extrasystoles, the extrasystolic beat would produce

decreased pulse pressure because stroke volume is decreased

a 24 year old woman presents to the er with sever diarrhea, when she is supine her bp is 90/60, and her HR is 100 bpm, when she is moved to a standing position her HR further increases to 120 bpm, which of the following accounts for the further increase in HR upon standing?

decreased venous return

propanolol has which of the following effects?

decreases HR

Acidosis

decreases K+ excretion and alkalosis increases K+ excretion

PR interval means

depolarization of AV node

QRS complex shows

depolarization of both ventricles

pulse pressure is

determined by stroke volume

Preload is end-

diastolic pressure (140 mL)

Preload can have up to 190 mL and so it increases

diastolic pressure and stoke volume

Pulse pressure

difference between systolic and diastolic

In ectopic foci the heart tissue receives signals from

different cells outside of the conductive system

ECG leads can detect

different functions of different parts of the heart

Which medication can cause arrhythmia?

digoxin (this medication decreases heart rate and increases contractility)

Vasodilaiton

dilaition of BVs dec BP

What is cardiomyopathy?

dilation of the heart

Rest of potassium is absorbed by

distal and collecting tubules

8% of Na+ is absorbed by

distal tubule and collecting tubule

When the case is renal disease go for

diuretic

What does Leukemia mean?

eak immune system because of immature WBCs

If there is lack of albumin in part of the body, there is risk for...

edema

The wall of artery has more

elastic fibers than veins

Volume that exists in ventricle is

end systolic volume (70 mL)

The amount of blood in the ventricle after the contraction of the the atrium is called

end-diastolic volume or preload

The remnant of blood which stays in ventricle after ejection of stoke volume is called

end-systolic volume

Obesity is associated with what kind of disorder?

endocrine disorder such as Type II diabetes

Hormonal changes due to pregnancy can affectD

endothelial layer of blood vessels and cause hypertension

capillaries are

epithelial cells surrounded by basal lamina site of exchange thin walled

The formed elements are classified as blood cells and are:

erythrocytes leukocytes Platelets

What does it mean to have a positive effect on cardiovascular system?

everything is increased, such as chronotropic (heart rate), dromotropic (conduction velocity), and inotropic (contractility of myocardium)

What is the long term effect of pulmonary hypertension?

excess blood in lung blood vessels release plasma into lung tissues

What does the irritation of beta-1 mean?

excess contractility of myocardium

polycythemia

excess of red blood cells

Systolic pressure

exists in ventricle during ventricular contraction

Lymphatic system collects what and release it where?

extra body fluids and releases it into venous sytem

aplastic anemia

failure of blood cell production in the bone marrow may be caused by chemicals or radiation

Deficiency of K+ means

failure of relaxation phase, which can cause arrhythmia

luekemia symptoms

fever and night sweats headache bleeding easily bone or joint pain swollen lymphnodes(armpit + neck) infections weakness weightloss

the physiological function of the relatively slow conduction through the atrioventricular node is to allow sufficient time for

filling of the ventricles

in a capillary, P(c) is 30 mm Hg, P(1) is 2mm Hg, x(c) is 25mm Hg, and x(1) is 2mm Hg, what is the direction of fluid movement and the net driving force?

filtration a 9mm Hg

The blood that goes into the kidney is needed for

filtration, urinary production, and excretion

Estrogen increases

fluid and NaCl production

How do you treat sickle cell anemia

folic acid supplements also antibiotics and vaccines are given to prevent bacterial infection

Luekopoiesis

formation of WBC's

What does hematopoiesis mean?

formation of blood stem cells

Medical problems caused by sickle cell anemia?

forming pigment gallstones jaundice painful episodes

Accumulation of Ca2+ leads to calcification which causes the blood vessels to become

fragile

the low resistance pathways between myocardial cells that allow for the spread of action potentials are the

gap junctions

Which arterys does filtration occur between?

glomerular capillary and Bowman's capsule

which of the following substaces crosses capillary walls primarily through water-filled clefts between the endothelial cells?

glucose

5 types of leukocytes, categorized by staining properties:

granulocytes- basophils, neutrophils, eosinophils (-phils = colorful) Agranulocytes- lymphocytes, monocytes (-cytes = not colorful)

Right after arch of aorta the descending aorta

has a smaller diameter is more narrow

Symptoms of hypertension?

headache, vomiting, nose bleeding, vision problems, and sleep disorders

Myeloid and lymphoid tissues are the site of what processes?

hematopoiesis, erythropoiesis, luekopoiesis (in bone marrow)

Normal hemoglobin is

hemoglobin A

Sickle cell anemia contains what type of hemoglobin?

hemoglobin S (abnormal, and so is hemoglobin C)

Excess and increased cortisol leads to

hyperglycemia, central obesity, weak immune system, and hypertension

Obesity causes

hyperlipidemia

Irritation/manipulation/tumor of sympathetic system leads to

hypertension

Polycystic kidney disease has

hypertension

Using cortisol for autoimmune disease, such as dermatitis, in long term can lead to

hypertension

Preeclampsia and eclampsia mean

hypertension during pregnancy

Deficiency of maternal thyroid hormones lead to

hypothyroidism

During appendicitis, the appendix gives a massive signal to the _____ _______ and causes ______ and _______ which ultimately blocks flow.

ileocecal sphincter, paralysis, spasm

White blood cells are considered either____ or _____

immature or mature

Where is the heart located?

in the thoracic cavity, between the two lungs

In womb hematopoesis first starts where?

in the yolk sack, then goes to the liver till brith

and increase in contractility is demonstrated on a frank-starling diagram by

increase cardiac output for a given end-diastolic volume

Cortisol leads to

increase in appetite and obesity

Excess aldosterone lead to

increased Na+, decreased K+

Increased aldosterone leads to

increased Na+, decreased K+, and severe hypertension

Having a tumor, infection, or autoimmune disease in adrenal medulla leads to

increased adrenal gland hormones and hypertension

What are the conditions that increase oxygen consumption?

increased afterload, increased size of heart, increased contractility and heart rate

Excess parathyroid hormones lead to

increased blood Ca2+, which causes hypercalcemia

Greater venous return leads to

increased cardiac output (5 L/min

Secretion of beta-1 causes

increased heart rate and contractility of myocardium

the change indicatied by the dashed lines on the cardiac output/neous return curves shows

increased mean systemic pressure

an ECG on a person shows ventricular extrasystoles, the next "normal ventricular contraction that occurs after the extrasystole would produce:

increased pulse pressure because the contractility of the ventricle is increased

which of the following chages will cause an increase in myocardial O2 consumption?

increased size of the heart

Increased contractility from myocardium results in

increased stoke volume and decreased end-systolic volume

the tendency for edema to occur will be increased by

increased venous pressure

The extra (140 mL) blood then causes tension which

increases the length of myocardial fibers

What is myocarditis?

inflammation of myocardium

What is the second function of ANP?

inhibits renin and aldosterone secretion

How can you measure filtration rate?

insulin

The other 10% of Plasma is

ions, nutrients, hormones, enzymes, antibodies, wastes and proteins (3 protein types)

Production of RBCs and hemoglobin is dependent on what 3 factors?

iron vitamin B12 folic acid

Duration of P wave

is 0.08 - 0.1 seconds

Platelets = Thrombocytes

-cell fragments from broken megakaryocytes -func in blood clotting -releases serotonin to constrict BV's at site of injury -Also secrete growth factors -LS:5-10 days

Where are the Kidneys located?

-in the upper abdominal quadrants of the retroperitonial space.

Diabetes

-insulin allows glucose to go into cells without it glucose will build up in the blood. -2 main types of diabetes ~Type 1: autoimmune disorder where immune system destroys beta cells so insulin can no longer be released. ~Type 2: results from insulin resistance, beta cells are fine insulin is present but the body has becomes resistant to insulin bc its been bombarded by glucose and insulin goes sky high then the pancreas is always working overtime and will not be able to keep up anymore. -Diabetes is 7th leading cause of death in the US.

Monocytes

-kidney shaped nucleus -precursor is monoblasts -develop into macrophages -LS is months -phagocytosis function -4-8% of WBCs -largest luekocyte

Eosinophils

-kill parasites -bilobed nucleus -life span: 8-12 days -bright red staining

Urinary Bladder

-made up of special epithelial cells that help[ allow for stretching of bladder ( Transitional epithilial cells) -1.5L-2L per day. - 2 different sphincters ~external we control ~internal -Feel full around 150-200 -sense of urgency around 300-400 ->600 involuntary urination

Albumin

-maintains osmotic pressure of blood

Nuetrophils

-phagocytize bacteria -life span 6 hrs-few days -stains pale red -multilobed

Albumin

-produce osmotic pressure(pulls water to capillaries) -60-80% plamsa protein -produced in liver

Distal Convoluted tubule

-reabsorption of sodium, bicarbonate, and secretion of ammonium, -PTH acts on DCT to stimulate calcium reabsorption -Impermeable to water -Empties into collecting duct

endocrine system

-regulates more slow processes

Basophils

-release histamine (inflammation) -function in allergic reactions -dark blue -bilobed -short LS

Luteinizing hormone (LH)

-targets ovaries and testes, in females it triggers ovulation in males it promotes testosterone production.

Follicle-stimulating hormone (FSH)

-targets ovaries and testes, in females stimulates ovary maturation and promotes estrogen, in males stimulates sperm production.

Thyroid stimulating hormone (TSH)

-targets thyroid to secrete T4 and T3

The fibrinogen plasma protein is...

-very important in blood clotting (injury) -made by liver

Lymphocytes

-very small, barely any cytoplasm (nucleus is majority of cell) -func in immune response -3 types (natural killer, t and b cells) -LS: hours to years -20-45% WBCs

The bundle of His has how many branches

2

2. The answer is D. After drinking distilled water, subject A will have an increase in intracellular fluid (ICF) and extracellular fluid (ECF) volumes, a decrease in plasma osmolarity, a suppression of antidiuretic hormone (ADH) secretion, and a positive free-water clearance (CH2O), and will produce dilute urine with a high flow rate. Subject B, after drinking the same volume of isotonic NaCl, will have an increase in ECF volume only and no change in plasma osmolarity. Because subject B's ADH will not be suppressed, he will have a higher urine osmolarity, a lower urine flow rate, and a lower CH2O than subject A.

2. Subjects A and B are 70-kg men. Subject A drinks 2 L of distilled water, and subject B drinks 2 L of isotonic NaCl. As a result of these ingestions, subject B will have a(A) greater change in intracellular fluid (ICF) volume(B) higher positive free-water clearance (CH2O)(C) greater change in plasma osmolarity(D) higher urine osmolarity(E) higher urine flow rate

When blood flow reaches capillary level, it drops to what mmHg?

20

When blood flow reaches to capillary level how many mmHgs is it?

20

What percent of our cardiac output from the heart do our kidneys receive?

20% (2nd most cardiac output following the liver)

20. The answer is B. Thiazide diuretics have a unique effect on the distal tubule; they increase Ca2+ reabsorption, thereby decreasing Ca2+ excretion and clearance. Because parathyroid hormone (PTH) increases Ca2+ reabsorption, the lack of PTH will cause an increase in Ca2+ clearance. Furosemide inhibits Na+ reabsorption in the thick ascending limb, and extracellular fluid (ECF) volume expansion inhibits Na+ reabsorption in the proximal tubule. At these sites, Ca2+ reabsorption is linked to Na+ reabsorption, and Ca2+ clearance would be increased. Because Mg2+ competes with Ca2+ for reabsorption in the thick ascending limb, hypermagnesemia will cause increased Ca2+ clearance.

20. Which of the following causes a decrease in renal Ca2+ clearance?(A) Hypoparathyroidism(B) Treatment with chlorothiazide(C) Treatment with furosemide(D) Extracellular fluid (ECF) volume expansion(E) Hypermagnesemia

22. The answer is B. This patient's plasma and urine osmolarity, taken together, are consistent with water deprivation. The plasma osmolarity is on the high side of normal, stimulating the posterior pituitary to secrete antidiuretic hormone (ADH). Secretion of ADH, in turn, acts on the collecting ducts to increase water reabsorption and produce hyperosmotic urine. Syndrome of inappropriate antidiuretic hormone (SIADH) would also produce hyperosmotic urine, but the plasma osmolarity would be lower than normal because of the excessive water retention. Central and nephrogenic diabetes insipidus and excessive water intake would all result in hyposmotic urine.

22. A woman has a plasma osmolarity of 300 mOsm/L and a urine osmolarity of 1200 mOsm/L. The correct diagnosis is(A) syndrome of inappropriate antidiuretic hormone (SIADH)(B) water deprivation(C) central diabetes insipidus(D) nephrogenic diabetes insipidus(E) drinking large volumes of distilled water

23. The answer is C. Effective renal plasma flow (RPF) is calculated from the clearance of para-aminohippuric acid (PAH) [CPAH = UPAH × V/PPAH = 600 mL/min].Renal blood flow (RBF) = RPF/1 - hematocrit = 1091 mL/min.

23. A patient is infused with paraaminohippuric acid (PAH) to measure renal blood flow (RBF). She has a urine flow rate of 1 mL/min, a plasma [PAH] of 1 mg/mL, a urine [PAH] of 600 mg/mL, and a hematocrit of 45%. What is her "effective" RBF?(A) 600 mL/min(B) 660 mL/min(C) 1091 mL/min(D) 1333 mL/min

HCO3- produced in the parietal cells is exchanged with ____. This bicarbonate is taken into the blood and delivered to the ______ where it is used to neutralize H+ in the small intestine.

Cl-, pancreas

What is the treatment for endocarditis?

Clean up bacteria for long-term, at least for 4-6 weeks, with strong antibiotic

What is your first heart sound caused by?

Closure of the AV valves (mitral and tricuspid)

What is your second heart sound caused by?

Closure of the semilunar valves (pulmonary and aortic)

How does blood protect?

Clotting- prevents blood loss while injured, prevents microbes from entering Immune- luekocytes fight infection

Aorta cannot carry normal amount of blood to lower limbs

Coarctation

Irritation of the cecum and distention of the cecum result in the production of the ______ reflex which _______ ileal sphincter tone and ______ ileal peristalsis.

Colonoileal, increases, decreases.

The opening for the ____ _____ ______ is found in the descending portion of the duodenum and is called the _____ ______ ______.

Common bile duct, major duodenal papilla

Pyelonephritis

Condition involving bacterial infection (mostly E.coli) in one or both kidneys. Causes: Advanced or untreated urinary tract infection (UTI) that has spread into ureters or renal system. Infection in the circulatory system that has entered the renal system through the renal artery. Symptoms: chills, fever, back pain, severe abdominal pain, nausea, vomiting, changes in color/scent of urine, fresh blood in urine (may appear pink) Treatment: Urinalysis and bloodwork is to confirm. One month of antibiotics.

Chorionic Metabolic Acidosis

Condition of metabolic acidosis in the chorion of a developing embryo.

Coarctation

Congenital problem

What are the causes of endocarditis?

Congenital, post-surgery, post-rheumatic heart disease

Blood

Connective tissue made of plasma, erythrocytes, leukocytes, and platelets. Blood cells: formed elements. 45% Plasma: fluid portion. 55%

Kidney (nephron)

Consists of glomerulus and renal tube. Blood enter each kidney via renal artery, branches into interlobar A, arcuate A, cortical radial. Second capillary network, the peritubular capillaries (surround nephrons), reabsorb solutes and water and flow into small veins. -Glomerulus: Glomerular capially network First network), emerges from afferent arteriole. -Renal tubule: Compromises the following: proximal tubule, loop of Henle (thin decending limb, thin/thick ascending limb), distal tubule, collecting ducts *In the juxtamedullary nephrons, the peritubular capillaries have a specialization called the vasa recta. Vasa recta serve as osmotic exchangers for the production of concentrated urine

What does the higher pressure in the atrium lead to?

Contraction and release of 70 mL into ventricle

Does the skeletal muscle in the vein contract or dilate?

Contracts, which in turn squeezes the vein

Pulmonary valve

Control blood flow from right ventricle to pulmonary arteries, which carry blood to lungs to pick up oxygen. Pulmonary artery: Carry blood to lungs to pick up oxygen Pulmonary vein: take blood from lungs to left atrium.

Treatments for mitral stenosis?

Control blood pressure Use anticoagulant (heparin) Use Digoxin (very important!!!)

Aortic valve

Controls flow of oxygen-rich blood from left ventricle to rest of body. Opens the way for oxygen rich blood to pass from left ventricle to aorta (largest artery)

What are the two internal parts of the kidney?

Cortex and medulla

What cranial nerves take information from the baroreceptors to the brainstem?

Cranial Nerves IX and X

What cranial nerves take the information and where is it taken?

Cranial nerves IX and X and they are taken to peripheral to hypothalamus

A patient is complaining of abdominal cramps, tenesmus, and weight loss. CBC results point to anemia. What is the correct diagnosis?

Crohn's disease

Patients with gastric ulcers have an increase in ______ secretion and a decrease in ______.

Gastrin, H+

Zollinger-Ellison syndrome occurs when __________ is secreted by non-beta cells of the _________.

Gastrin, pancreas

A patient complains of black stool, vomiting blood, and loss of appetite. What is the correct diagnosis?

Gastritis

Causes of myocardial infarction

Genetic Hypertension Atherosclerosis Hyperlipidemia Stress Endocrine hormone disorders (type I and II diabetes) (excess glucose destroys walls of blood vessels)

____________ is secreted by gastric cells and stimulates orexigenic neurons while inhibiting anorexigenic neurons, causing increased appetite.

Ghrelin

What do lymphoid cells do?

Give rise to B lymphocytes and T lymphocytes

What do Myeloid cells do?

Give rise to erythrocyte, eosinophil, basophil, neutrophil, monocyte, and platelet

Myeloid

Gives rise to erythrocytes, basophils, eosinophils, neutrophils, and monocytes.

Due to the increased length of muscle fiber created from the tension of the 140 mL of blood there are 2 factors. What is the second one?

Increases sensitivity of troponin C to Ca2+, which leads to binding of Ca2+ to troponin C and strong muscle contraction by interweaving of actin and myosin

Parallel resistance

Illustrated by systemic circulation. Smaller diameter means higher resistance.

when propranolol is administered, blockade of which receptor is responsible for the decrease in cardiac output that occurs?

it inhibits Beta 1 receptors in the SA node(HR) and in ventricular muscle

Why is blood volume also stress volume

it is under high pressure in artery since this pressure comes from strong contraction of ventricle

Why is the myocardium full of mitochondria?

it needs ATP,

High level of bilirubin lead to?

jaundice (yellowish color of skin)

Symptoms for endocarditis if it is bacterial?

joint pain, fever, weakness, abnormal urine color and smell

Receptive relaxation is a ______ reflex mediated by the _____ nerve in the ______ region of the stomach in the presence of the hormone _______.

vasovagal, vagus, orad, CCK

V = Q/A

velocity equals blood flow divided by cross sectional area V inversely proportional to cross section V directly proportional to Blood flow

70 mL goes from the atrium to

ventricle

Action potential in ____,_____,and ____ has similar steps? (5 steps)

ventricle, atrium, and purkinje fibers

Diastolic pressure

ventricles during relaxation phase of ventricle

Where does the pressure from afterload come from?

ventricular contraction

What are sinusoids?

weak leaky capillaries found in the liver and spleen

Deficiency of thyroid hormone can lead to

weight gain (obesity), memory/learning disorder, and hypotension

Excess thyroid hormone can lead to

weight loss, hypertension, palpitations, and sweating

Stimulation of sympathetic system will lead to

which causes vasoconstriction → increases total peripheral resistance (TPR) → builds up pressure and blood supply

Relative Refractory Period (RRP)

Immediately after ARP when repolarization is almost complete. It is possible to generate a second action potential. Action potential can be elicited, but more than the usual inward current in required.

What does gamma globulin do?

Important for protection from viruses

Kidney stones (renal calculi)

"Calculi". Hard masses formed within the urinary tract. Result of highly concentration urine saturated with salts. Accumulates over time. Due to deficiency of inhibitory factors (citrate). Stone composition: ~80% calcium, and varied amounts of urine acid, cysteine amino acid. More common with hyperparathyroidism (parathyroid controls secretion of calcium into the blood). Stone formation can obstruct urinary tracts and cause a decrease in urination and increase in fluid retention. Damages tissues and susceptible to bacterial infection. Treatment: If stones are small, may pass on its own. Increase in water consumption. Surgery. Pain can be alleviated with non-steroidal anti-inflammatory drugs (NSAIDs) or morphine. -Verapamil: acts as a calcium ion channel blocker to inhibit the passage of calcium into bloodstream. -Potassium citrate: supplemental citrate salt that dissociates from potassium in the body and attaches to calcium to inhibit stone formation via calcification. -Tamulosin: alpha-adrenergic receptor blocker which target cells with this receptor type, resulting in smooth muscle relaxation of bladder neck so urine and stone can pass through.

Hyperaldonsteronism

"Conn's disease". Tumors on cells specific for aldosterone secretion in adrenal cortex. Symptoms: hypertension, headache, vomiting, nausea, sleep disorder, nose bleeds, hypernatremia, hypoalkemia. Treatment: Surgical removal of tumor. Medications to control symptoms.

Steps 1-2: Isovolumetric contraction

"Early ventricular systole" Begins during diastole at point 1. Left ventricle is filled from left atrium (volume: ~140 mL, end-diastolic volume). Ventricular pressure is low (relaxed). On excitation, ventricle contracts and pressure increases. Mitral valve closes when left ventricular pressure is greater than left atrial pressure. No blood can be ejected.

Step 2-3: Ventricular systole (ejection)

"Ejection period". Aortic valve opens at point 2 when pressure in the left ventricle exceeds pressure in the aorta. Blood is ejected into aorta and ventricular volume decreases. Known as the stroke volume. The volume remaining in left ventricle at point 3 is end-systolic volume.

Primary hypertension

"Essential hypertension". Has unknown cause. Environmental factors including excessive dietary sodium (increases intracellular calcium), obesity and stress (can affect hormone levels: cortisol raises blood pressure). An increase norephinpherine causes vasocontriction, decreases vasodilation hormones. Also include: hormonal imbalances, renal dysfunction, cardiovascular issues, obstruction or destruction of blood vessels, mechanism of sympathetic nervous system, and renin-angiotensin-aldosterone mechanisms. Can increase cardiac output and increase total peripheral resistance (TPR) Sodium: Abnormal Na+ transport across the cell wall due to a defect in or inhibition of the sodium-potassium pump or because of increased permeability to the Na+. Increased intracellular Na+, which makes the cell more sensitive to sympathetic stimulation. Since Ca2+ follows Na+, accumulation of intracellular Ca2+ is responsible for the increased sensitivity. Deficiency of a vasodilator substance: prostaglandin, bradykinin

Glomerulonephritis

"Nephritic syndrome". Disorder characterized by localized edema and inflammation of the glomeruli, high blood pressure and blood in urine. Causes: Differ depending on acute or chronic. Symptoms: Edema (seen early on) which can result in vasculitis of the glomerulus capillaries (damage that decreases their filtration rate). Decrease urine production. Can lead to obstruction of blood-flow and fluid retention in other areas of the body. Edema may progress to legs. High blood pressure, hypertension, headache, fatigue, vomiting, nausea, vertigo, nosebleeds, fever, weakness, malaise, coma, loss of appetite, abdominal pain, visual disturbances, joint pain. -If cause is autoimmune, leukocytes may infiltrate abdominal tissues and cause inflammation in GI, joints and other visceral organs. Resulting in joint pain, abdominal pain, loss of appetite. Treatment: Bloodwork and urinalysis to determine severity and cause. Low protein, low sodium diet if less severe. Diuretics to increase urine production and flush retained fluid and solute (sodium). Angiotensis converting enzyme (ACE) inhibitor and Angiotensin II receptor blockers (ARBs) for controlling hypertension and high blood pressure. Antibiotics for bacterial infection. If renal failure, dialysis is recommended. Two degrees of severity: Primary and Secondary.

Polycythemia Vera (PV)

"Polyvera". Abnormal excess in erythrocyte concentration resulting from two causes: -Primary: caused by an abnormality in bone marrow due to a tumorous growth, genetic condition, or congenital condition. -Secondary: Caused by excessive production of erythropoietin hormone signal which can be induced artificially by intravenous "blood-doping" Signs and Symptoms: Increased levels of erythrocytes, leukocytes and platelets, and increased hematocrit. Headaches, hypertension, vomiting and difficulty sleeping.

Lymphocytic leukemia

"lymphoblastic" produces too much mature lymphocytes

Causes of secondary hypertension

(1) Hormonal disorder (2) Medication (side effects) 3) Cardiovascular 4) Renal system 5) Physiologic - Pregnancy can cause preeclampsia/eclampsia 6) Congenital problems

Functions of the Kidney?

- Right Kidney (L1-L3) Left Kidney (T12-L3) - Regulation of volume, composition, and pH of body fluid. - Regulation of acid-base homeostasis (via production of ammonia) - Regulation of energy metabolism via gluconeogenisis (the formation of new glucose from non carbohydrate sources) during fasting conditions -Regulation of plasma osmolarity through the control of aquaporin receptors within the collecting duct. -Detoxification of metabolic waste through excretory mechanisms. -Conversion of vitamin D3 to its active form. (125 dihydroxyndbweri) -Synthesis and conversion of important hormones such as erythropoietin (Maintains and regulates hematopoiesis in bone marrow) and renin (plays major role in renin-angiotensin-aldosterone system RAAS to help maintain blood pressure).

More on blood supply

-(cortical radial =interlobular ) -peritubular capillaries (PCT DCT) -vass recta (loop of henley) -afferent arteriole (glomarulous)

Hemoglobin is a protein which has...

-4 protein chains all attached to one heme -1 heme(iron containing+ red pigment)

Spike potentials are triggered at ______mV.

-40

Adrenal gland (supra renal glands)

-Adrenal hormones help us cope with stressful circumstances. -adrenal medulla: part of sympathetic nervouse system Adrenal cortex: 2 dozen steroid hormones called corticosteroids.

Hormones

-Aldosterone: ~Promotes sodium reabsorption in the DCT and collecting duct ~Promotes retention of water and sodium -Antidiuretic hormone/ vasopressin ~Inserts more aquaporins which increases water reabsorption, so increase of blood volume so increase bloop pressure.

Two broad molecular categories

-Amino acid based ~water soluble, cannot cross plasma membrane ~receptors usually on surface of cell -Steroid based ~lipid soluble ~receptors are on the inside of the cell

Pancreas

-Both endocrine and exocrine functions. -Exocrine: digestive system and digestive enzymes -Endocrine: islets of langerhans -alpha and beta ~alpha -glucagon-> raises blood glucose levels. (hypogycimia) ~beta -insulin->lowers blood glucose (hyperglycemia)

Thyroid gland

-Butterfly shaped gland, Two lobes. -Composed of hollow spherical follicles -Produces Thyroid hormone: ~T4 four iodines and T3 three iodines ~effects every cell in the body ~can go inside the cell ~excess thyroid hormone: thin, hot ~minimal thyroid hormone: sluggish, overweight, cold. -The thyroid is the only endocrine gland that stores its hormones extra cellularly in large quantities. -thyroid hormone: Follicular cells -Calcitonin: parafollicular cells

Renal Calculi (kidney stones)

-Calcium oxalate most common -Severe pain in the back -Dehydration causes bc then solvents come together. -Lithotripsy uses shock waves to break up stones.

Structure of heart

-Located in mediastinum. Between 1st and 5th intercostal spaces. -Four chambers - left atrium and ventricle (largest chamber, has enough force to push blood through aortic valve into entire body), right atrium and ventricle separated by septum (wall of muscle) -Two atrioventricular valves (tricuspid, mitral), two semilunar (pulmonary, aortic) valves -Three layers - epicardium, myocardium, endocardium -The two AV valves are located at the entrance into the ventricles. They are called the tricuspid valve and the bicuspid (mitral) valve. The tricuspid valve is located between the right atrium and the right ventricle; the bicuspid (mitral) valve is located between the left atrium and the left ventricle. -The semilunar valves are located at the exit of each ventricle at the beginning of the great vessels. They are known as the pulmonic valve and the aortic valve. The pulmonic valve is located at the entrance of the pulmonary artery as it exits the right ventricle. The aortic valve is located at the beginning of the ascending aorta as it exits the left ventricle.

Nephrons

-Main functional unit of kidney, responsible for urine formation. -Two types termed based on location ~Cortical [80-85%] (majority of it renal corpuscle found in cortex) >shorter loops of henley >more diluted urine ~Juxtamedullary [15-20%] (renal corpuscle adjacent to medulla) >longer loops of henley >more concentrated urine *Loops of henley (play a big role in water reabsorption) -Composed of: ~Renal corpuscle (bowman capsule and glomarulis) ~Renal Tubule (PCT Proximal convoluted tubule, Loop of henley , DCT distal convoluted tubule ~Collecting duct

Kidney

-Maintains constancy of ECF volume and osmolality by balancing intake and excretion of sodium and water. -Achieves constancy of extracellular potassium concentration, blood, and cellular pH by adjusting H+ and HCO3. -Conserves nutrients: glucose, amino acids. -Excretes end products of metabolism: urea, uric acid. The fraction that is not reabsorbed remains in tubules and appears in terminal urine. -Metabolic functions arginine formation, gluconeogensis, peptide hydrolysis. -Source of hormones: angiotensin 2, erythropoietin, prostaglandins -Reabsorption: Greater part of ultrafiltrate is transported across tubule wall and reenters blood. -Secretion: Urinary solvents enter nephron lumen from tubule.

UTI

-More in sexually active bc allows for transfer of bacteria -more in women bc shorter urethra - Symptoms ~burning ~increased urgency and frequency of micturition ~fever ~sometimes cloudy or blood tinged urine -UTI in elderly can include mental changes and confusion.

PTH vs Calcitonin

-PTH: raise blood calcium levels -Calcitonin: Decrease blood calcium levels.

Cerebral Ischemia

-Pco2 increases in brain tissue. Chemoreceptors in the vasomotor center respond by increasing sympathetic outflow to heart and blood vessels. Constriction of arterioles causes intense peripheral vasoconstriction and increased TPR. Blood flow to other organs (kidneys) is significantly reduced in an attempt to preserve blood flow to brain. Ex: Cushing reaction: Increases in intracranial pressure can cause compression of cerebral blood vessels. Vasomotor directs an increase in sympathetic outflow to the heart and blood vessels which increases arterial pressure.

Blood contents

-Plasma 55% (mainly water) >Albumin- Osmotic pressure >Gloubulin- transports lipids and fat-soluble vitamins >Fibrinogen-clotting -Formed Elements 45% >

Length tension: ventricles

-Preload: Equivalent to end diastolic volume (related to right atrial pressure). Venous return increases, end-diastolic volume increases, stretches or lengthens the ventricular muscle fibers. Frank-Starling relationship. Increased preload: Increase in end-diastolic volume, result of increased venous return. -Afterload: Left ventricle is equivalent to aortic pressure. Increases in aortic pressure cause an increase in afterload on the left ventricle. Right ventricle is equivalent to pulmonary artery pressure. Can increase afterload on right ventricle. Increase afterload: Increase in aortic pressure. Decrease in stroke volume. Increase in end-systolic volume. -Sarcomere length: Determines maximum number of cross-bridges that can form between actin and myosin. Determines maximum tension (force of contraction) -Velocity of contraction at a fixed muscle length. Maximal when the afterload is zero. Decreased by increases in afterload. Increased contractility: Ventricle has greater tension than usual during systole causing an increase in stroke volume. Decrease in end-systolic volume.

Pineal gland

-Secretes melatonin, which regulates our ciradian rhythm. -melatonin release is stimulated by darkness. ~Blue light blocks melatonin secretion. -highly vascular structure.

Urinary system is constantly working to maintain;?

-The purity and health of the body's fluids by removing unwanted substances and recycling others. -(our body is going to be trying to recycle proteins and nucleic acids etc.)

Loop of henley

-Thin decsending limb ~ only permeable to water (passive transport) -Thin ascending limb -Thick ascending limb ~impermeable to water, active transport of sodium which drives transport of chloride ions. -Interstitial fluid is Hypertonic to the filtrate.

Kidneys also excrete:?

-Toxins -Excess ions like sodium and potassium, reabsorption of calcium -Water

Alpha and beta globulins have same fuction:

-Transport lipids and fat soluble protein -funciton in immunity -produced in liver

What are the functions of our blood?

-Transportation: Blood contents, gases, nutrients. -Regulation: Hormones and temperature. -Protection: white blood cells (immune system), platelets (clotting).

Glomalular filtration rate

-Volume of filtrate produced by both kidneys per minute.

Collecting duct

-Water reabsorption back into the blood. -Contains aquaporins -sodium reabsorption and potassium excretion.

Corticosteroids

-Zona glomerulosa: aldosterone, control mineral and water in blood Fasciculota: glucocorticoids like cortisol Reticularis: sex steroids

How do the kidneys contribute to homeostasis?

-by regulating plasma composition through the elimination of metabolic wastes. ~3 main metabolic waste products (nitrogenous compounds) >Urea- formed by breakdown of amino acids (proteins) >Uric acid- formed by biproduct of breakdown of nucleic acids >creatnin- breakdown of creatin phosphate (molecule found in muscle tissues that helps aid in a storage of ATP).

Regulation of reabsorption of filtered bicarbonate

1.Lumen of proximal tubule contains urine 2.During cell respiration, the cell releases CO2 and this CO2 combines with H2O to form carbonic acid (H2CO3-) 3.Carbonic acid releases H+ and bicarbonate (HCO3-) 4.Bicarbonate (HCO3-) comes back to blood (capillary) by Na+ symport 5.H+ is excreted into urine (lumen) 6.Few bicarbonate (HCO3-) gets into lumen or is excreted 7.Bicarbonate (HCO3-) and H+ combine to form carbonic acid (H2CO3-) 8.Then carbonic acid (H2CO3-) releases H2O, which remains inside urine or is added in urine and CO2 comes back into epithelial cell for future bicarbonate (HCO3-) and H+ ion release 9.Having excretion of H+ ion makes the urine pH 4.4 (acidic) 10.Based on this mechanism in any cell (not just for kidney), excess CO2 increases H+ ion level (acidosis) More CO2, more acid Results in hyperventilation to decrease acid and CO2 level Acidemia causes hyperventilation 11.When bicarbonate (HCO3-) is high and H+ ion is low, it means alkalosis Alkalemia causes hypoventilation because do not need much ventilation

Renin-angiotensin-aldosterone cycle

1.Renin converts angiotensinogen into angiotensin I (AgI) 2.Angiotensin converting enzyme (ACE) converts AgI into AgII 3.AgII, itself, acts as a vasoconstrictor and it increases blood pressure 4. It stimulates aldosterone secretion 5.Blood then carries aldosterone to the nephron in kidney 6.Aldosterone binds to its receptor on nephron and stimulates capillary on nephron for absorption of Na+, Cl-, and H2O, which increases blood pressure and blood Na+ 7.Also excretes K+ into urine By this way, aldosterone increases blood pressure and blood Na+

10. The answer is D [III B; Figure 5-5]. At concentrations greater than at the transport maximum (Tm) for glucose, the carriers are saturated so that the reabsorption rate no longer matches the filtration rate. The difference is excreted in the urine. As the plasma glucose concentration increases, the excretion of glucose increases. When it is greater than the Tm, the renal vein glucose concentration will be less than the renal artery concentration because some glucose is being excreted in urine and therefore is not returned to the blood. The clearance of glucose is zero at concentrations lower than at Tm (or lower than threshold) when all of the filtered glucose is reabsorbed, but is greater than zero at concentrations greater than Tm

10. At plasma concentrations of glucose higher than occur at transport maximum (Tm), the(A) clearance of glucose is zero(B) excretion rate of glucose equals the filtration rate of glucose(C) reabsorption rate of glucose equals the filtration rate of glucose(D) excretion rate of glucose increases with increasing plasma glucose concentrations(E) renal vein glucose concentration equals the renal artery glucose concentration

Pressure that exists in aorta is about how many mmHg?

100

Pressure that exists in aorta is what mmHg?

100

Loop of Henle osmolarity?

100 mOsm/L

The volume of blood that circulates in arterial system is under pressure which is about

100 mmHgs

67% of Na+ can absorb what percentage of glucose?

100%

cardiac output on the right side of the heart is what percentage of the cardiac output of the left side of the heart?

100%

What is the lifespan of RBCs

100-120 days

What is the range of BPM for atrial tachycardia?

100-250 BPM

Life span and shape of RBC?

100ish days, biconcave

11. The answer is D. A person who produces hyperosmotic urine (1000 mOsm/L) will have a negative free-water clearance (-CH2O) [CH2O = V - Cosm]. All of the others will have a positive CH2O because they are producing hyposmotic urine as a result of the suppression of antidiuretic hormone (ADH) by water drinking, central diabetes insipidus, or nephrogenic diabetes insipidus.

11. A negative free-water clearance (-CH2O) will occur in a person who(A) drinks 2 L of distilled water in 30 minutes(B) begins excreting large volumes of urine with an osmolarity of 100 mOsm/L after a severe head injury(C) is receiving lithium treatment for depression, and has polyuria that is unresponsive to the administration of antidiuretic hormone (ADH)(D) has an oat cell carcinoma of the lung, and excretes urine with an osmolarity of 1000 mOsm/L

13. The answer is A. Increasing filtration fraction means that a larger portion of the renal plasma flow (RPF) is filtered across the glomerular capillaries. This increased flow causes an increase in the protein concentration and oncotic pressure of the blood leaving the glomerular capillaries. This blood becomes the peritubular capillary blood supply. The increased oncotic pressure in the peritubular capillary blood is a driving force favoring reabsorption in the proximal tubule. Extracellular fluid (ECF) volume expansion, decreased peritubular capillary protein concentration, and increased peritubular capillary hydrostatic pressure all inhibit proximal reabsorption. Oxygen deprivation would also inhibit reabsorption by stopping the Na+-K+ pump in the basolateral membranes.

13. Which of the following would produce an increase in the reabsorption of isosmotic fluid in the proximal tubule?(A) Increased filtration fraction(B) Extracellular fluid (ECF) volume expansion(C) Decreased peritubular capillary protein concentration(D) Increased peritubular capillary hydrostatic pressure(E) Oxygen deprivation

14. The answer is E. Interstitial fluid volume is measured indirectly by determining the difference between extracellular fluid (ECF) volume and plasma volume. Inulin, a large fructose polymer that is restricted to the extracellular space, is a marker for ECF volume. Radioactive albumin is a marker for plasma volume.

14. Which of the following substances or combinations of substances could be used to measure interstitial fluid volume?(A) Mannitol(B) D2O alone(C) Evans blue(D) Inulin and D2O(E) Inulin and radioactive albumin

Since the ventricle already has 70 mL, the total with the blood from the atrium would be...

140 mL (end-diastolic volume)

The ventricle, after the contraction of the atrium, has how many mL of blood in it?

140 mL of blood

How much blood does the cerebral system receive

15%

15. The answer is D. At plasma concentrations that are lower than at the transport maximum (Tm) for para-aminohippuric acid (PAH) secretion, PAH concentration inthe renal vein is nearly zero because the sum of filtration plus secretion removes virtually all PAH from the renal plasma. Thus, the PAH concentration in the renal vein is less than that in the renal artery because most of the PAH entering the kidney is excreted in urine. PAH clearance is greater than inulin clearance because PAH is filtered and secreted; inulin is only filtered.

15. At plasma para-aminohippuric acid (PAH) concentrations below the transport maximum (Tm), PAH(A) reabsorption is not saturated(B) clearance equals inulin clearance(C) secretion rate equals PAH excretion rate(D) concentration in the renal vein is close to zero(E) concentration in the renal vein equals PAH concentration in the renal artery

16. The answer is E. The person with water deprivation will have a higher plasma osmolarity and higher circulating levels of antidiuretic hormone (ADH). These effects will increase the rate of H2O reabsorption in the collecting ducts and create anegative free-water clearance (-CH2O). Tubular fluid/plasma (TF/P) osmolarity in the proximal tubule is not affected by ADH.

16. Compared with a person who ingests 2 L of distilled water, a person with water deprivation will have a(A) higher free-water clearance (CH2O)(B) lower plasma osmolarity(C) lower circulating level of antidiuretic hormone (ADH)(D) higher tubular fluid/plasma (TF/P) osmolarity in the proximal tubule(E) higher rate of H2O reabsorption in the collecting ducts

17. The answer is C. Dilation of the afferent arteriole will increase both renal plasma flow (RPF) [because renal vascular resistance is decreased] and glomerular filtration rate (GFR) [because glomerular capillary hydrostatic pressure is increased]. Dilation of the efferent arteriole will increase RPF, but decrease GFR. Constriction of the efferent arteriole will decrease RPF (due to increased renal vascular resistance) and increase GFR. Both hyperproteinemia (↑ π in the glomerular capillaries) and a ureteral stone (↑ hydrostatic pressure in Bowman's space) will oppose filtration and decrease GFR.

17. Which of the following would cause an increase in both glomerular filtration rate (GFR) and renal plasma flow (RPF)?(A) Hyperproteinemia(B) A ureteral stone(C) Dilation of the afferent arteriole(D) Dilation of the efferent arteriole(E) Constriction of the efferent arteriole

19. The answer is C. Both individuals will have hyperosmotic urine, a negative free-water clearance (-CH2O), a normal corticopapillary gradient, and high circulating levels of antidiuretic hormone (ADH). The person with water deprivation will have a high plasma osmolarity, and the person with syndrome of inappropriate antidiuretic hormone (SIADH) will have a low plasma osmolarity (because of dilution by the inappropriate water reabsorption).

19. Which of the following would best distinguish an otherwise healthy person with severe water deprivation from a person with the syndrome of inappropriate antidiuretic hormone (SIADH)?(A) Free-water clearance (CH2O)(B) Urine osmolarity(C) Plasma osmolarity(D) Circulating levels of antidiuretic hormone (ADH)(E) Corticopapillary osmotic gradient

Excitation-Contraction coupling steps

1: Action potential from cell membrane to T tubules. 2: During plateau of action potential, calcium conductance is increased and enters the cell from extracellular fluid (inward calcium current) through L-type Ca2+ channels (dihydropyridine receptors) 3: Triggers the release of calcium from SR through Ca2+ release channels (ryanodine receptors) 4: Intracellular calcium increases. 5: Calcium binds to troponin C and tropomyosin is moved out of the way, removing the inhibition of actin and myosin binding 6: Actin and myosin bind, thick and thin filaments slide past and the myocardial cell contracts. The magnitude that develops is proportional to the intracellular calcium. 7: Relaxation occurs when calcium is reaccumulated by the SR by an active calcium ATPase pump.

Baroreceptor steps

1: Decrease in arterial pressure, decreases stretch on the walls of cartoid sinus. 2: Decreases firing rate of cartoid sinus carnial nerve IX, carries information to vasomotor center in the brain. 3: Set point for arterial pressure in vasomotor is 100 mmHg. Series of autonomic responses is coordinated by vasomotor center if drops below set point. Will attempt to increase blood pressure to the norm. 4: Responses of vasomotor center to a decrease are coordinated to increase back to 100 mmHg. Decrease parasympathetic (vagal) outflow to the heart and increased sympathetic outflow to heart and blood vessels. ex: Response to acute blood loss

Steps in Renin-angiotensin-aldosterone system

1: Decrease in renal perfusion pressure causes the juxtaglomerular cells of the afferent arteriole to secrete renin. 2: Renin is an enzyme that catalyzes the conversion of angiotensinogen to angiotensin I in plasma. 3: Angiotensin-converting enzyme (ACE) catalyzes the conversion of angiotensin I to angiotensin II, primarily in the lungs. ACE inhibitors (ex: captopril) block the conversion of angiotensin I to angiotensin II, therefore, decrease blood pressure. Angiotensin receptor (AT1) antagonists (ex: losartan) block the action of angiotensin II at its receptor and decrease blood pressure.

Four attempts for arterial pressure

1: Increase heart rate 2: Increase contractility and stroke volume 3: Increase vasoconstriction of arterioles 4: Increase vasoconstriction of veins. Additional attempts: 1: Cerebral ischemia 2: Chemoreceptors in cartoid and aortic bodies 3: Vasaopressin (antidiuretic hormone) 4: Atrial natriuretic peptide (ANP)

Excretion (renal)

1: Plasma concentrations less than 250 mg/dL, all of the filtered glucose is reabsorbed and excretion is zero. Threshold is approximately 250mg/dL. 2: Plasma concentrations greater than 350 mg/dl, reabsorption is saturated ™. As the plasma concentration increases, the additional filtered glucose cannot be reabsorbed and is excreted in the urine.

Potassium regulation

1: Shifts of potassium between the ICF and ECF: Most of the body's potassium is located in the ICF. A shift of K+ out of cells causes hyperkalemia. A shift of K+ into cells causes hypokalemia. 2: Renal regulation of potassium balance: K+ is filtered, reabsorbed, and secreted by the nephron. K+ balance is achieved when urinary excretion of equals intake of K+ in the diet. K+ excretion can vary widely from 1% to 110% of the filtered load, depending on dietary K+ intake, aldosterone levels, and acid-base status. 3: Glumerular capillaries: Filtration occurs freely across the glomerular capillaries. 4: Proximal tubule: reabsorbs 67% of the filtered K+ along with Na+ and H2O. 5: Thick ascending limb of the loop of henle: reabsorbs 20% of the filtered K+. Reabsorption involves the Na+-K+-2Cl- cotransporter in the luminal membrane of cells in the thick ascending limb. 6: Distal tubule and collecting duct: either reabsorb or secrete K+, depending on dietary K+ intake

Reabsoprtion (renal)

1: Sodium glucose cotransport in proximal tubule reabsorbs glucose from tubular fluid into blood. Sodium glucose carriers are limited. 2: Plasma glucose concentration less than 250 mg/dL, all filtered glucose can be reabsorbed (plenty carrier available). Reabsorption line is the same as filtration 3: Plasma glucose concentration greater than 350 mg/dL, carriers are saturated. Does not increase rate of reabsorption. Transport maximum.

on the graph showing left ventricular volume and pressure, isovolumetric contraction occurs from point;

1>2

When is folic acid supplement important?

1st trimester in pregnancy

How many papillary muscles in left ventricle?

2

25. The answer is D. By sweating and then replacing all volume by drinking H2O, the woman has a net loss of NaCl without a net loss of H2O. Therefore, her extracellular and plasma osmolarity will be decreased, and as a result, water will flow from extracellular fluid (ECF) to intracellular fluid (ICF). The intracellular osmolarity will also be decreased after the shift of water. Total body water (TBW) will be unchanged because the woman replaced all volume lost in sweat by drinking water. Hematocrit will be increased because of the shift of water from ECF to ICF and the shift of water into the red blood cells (RBCs), which causes their volume to increase.

25. A woman runs a marathon in 90°F weather and replaces all volume lost in sweat by drinking distilled water. After the marathon, she will have(A) decreased total body water (TBW)(B) decreased hematocrit(C) decreased intracellular fluid (ICF) volume(D) decreased plasma osmolarity(E) increased intracellular osmolarity

26. The answer is A. Exercise causes a shift of K+ from cells into blood. The result is hyperkalemia. Hyposmolarity, insulin, β-agonists, and alkalosis cause a shift of K+ from blood into cells. The result is hypokalemia. 26. Which of the following causes hyperkalemia?(A) Exercise(B) Alkalosis(C) Insulin injection(D) Decreased serum osmolarity(E) Treatment with β-agonists

26. Which of the following causes hyperkalemia?(A) Exercise(B) Alkalosis(C) Insulin injection(D) Decreased serum osmolarity(E) Treatment with β-agonists

27. The answer is E. A cause of metabolic alkalosis is hyperaldosteronism; increased aldosterone levels cause increased H+ secretion by the distal tubule and increased reabsorption of "new" HCO3-. Diarrhea causes loss of HCO3- from the gastrointestinal (GI)tract and acetazolamide causes loss of HCO3- in the urine, both resulting in hyperchloremic metabolic acidosis with normal anion gap. Ingestion of ethylene glycol and salicylate poisoning leads to metabolic acidosis with increased anion gap.

27. Which of the following is a cause of metabolic alkalosis?(A) Diarrhea(B) Chronic renal failure(C) Ethylene glycol ingestion(D) Treatment with acetazolamide(E) Hyperaldosteronism(F) Salicylate poisoning

28. The answer is A. Parathyroid hormone (PTH) acts on the renal tubule by stimulating adenyl cyclase and generating cyclic adenosine monophosphate (cAMP). The major actions of the hormone are inhibition of phosphate reabsorption in the proximal tubule, stimulation of Ca2+ reabsorption in the distal tubule, and stimulation of 1,25- dihydroxycholecalciferol production. PTH does not alter the renal handling of K+.

28. Which of the following is an action of parathyroid hormone (PTH) on the renal tubule?(A) Stimulation of adenylate cyclase(B) Inhibition of distal tubule K+ secretion(C) Inhibition of distal tubule Ca2+ reabsorption(D) Stimulation of proximal tubular phosphate reabsorption(E) Inhibition of production of 1,25-dihydroxycholecalciferol

How much hemoglobin is in one erythrocyte?

280 hemoglobin

29. The answer is C. Hypertension, hypokalemia, metabolic alkalosis, elevated serum aldosterone, and decreased plasma renin activity are all consistent with a primary hyperaldosteronism (e.g., Conn's syndrome). High levels of aldosterone cause increased Na+ reabsorption (leading to increased blood pressure), increased K+ secretion (leading to hypokalemia), and increased H+ secretion (leading to metabolic alkalosis). In Conn's syndrome, the increased blood pressure causes an increase in renal perfusion pressure, which inhibits renin secretion. Neither Cushing's syndrome nor Cushing's disease is a possible cause of this patient's hypertension because serum cortisol and adrenocorticotropic hormone (ACTH) levels are normal. Renal artery stenosis causes hypertension that is characterized by increased plasma renin activity. Pheochromocytoma is ruled out by the normal urinary excretion of vanillylmandelic acid (VMA).

29. A man presents with hypertension and hypokalemia. Measurement of his arterial blood gases reveals a pH of 7.5 and a calculated HCO3-of 32 mEq/L. His serum cortisol and urinary vanillylmandelic acid (VMA) are normal, his serum aldosterone is increased, and his plasma renin activity is decreased. Which of the following is the most likely cause of his hypertension?(A) Cushing's syndrome(B) Cushing's disease(C) Conn's syndrome(D) Renal artery stenosis(E) Pheochromocytoma

What month does liver and spleen take over for hematopoiesis of the fetus?

2nd

How many papillary muscles in right ventricle?

3

the aortic valve closes at point :

3

Slow waves in the caudad region of the stomach occur at a frequency of ___ to ____ per minute.

3, 5

The transit time from the pylorus to the ileocecal valve is ____ to _____ hours.

3, 5

3. The answer is A. An acid pH, together with decreased HCO3- and decreased PCO2, is consistent with metabolic acidosis with respiratorycompensation (hyperventilation). Diarrhea causes gastrointestinal (GI) loss of HCO3-, creating a metabolic acidosis.

3. The correct diagnosis for this patient is (A) metabolic acidosis(B) metabolic alkalosis(C) respiratory acidosis(D) respiratory alkalosis(E) normal acid-base statu

30. The answer is D. The history strongly suggests chronic obstructive pulmonary disease (COPD) as a cause of respiratory acidosis. Because of the COPD, the ventilation rate is decreased and CO2 is retained. The [H+] and [HCO3-] are increased by mass action. The [HCO3-] is further increased by renal compensation for respiratory acidosis (increased HCO3- reabsorption by the kidney is facilitated by the high PCO2).

30. Which set of arterial blood values describes a heavy smoker with a history of emphysema and chronic bronchitis who is becoming increasingly somnolent?

Distal tube

300 mOsm/L

What is the urine osmolarity in proximal tubule?

300 mOsm/L

31. The answer is B. The blood values in respiratory alkalosis show decreased PCO2 (the cause) and decreased [H+] and [HCO3-] by mass action. The [HCO3-] is further decreased by renal compensation for chronic respiratory alkalosis (decreased HCO3- reabsorption).

31. Which set of arterial blood values describes a patient with partially compensated respiratory alkalosis after 1 month on a mechanical ventilator?

32. The answer is E. In patients who have chronic renal failure and ingest normal amounts of protein, fixed acids will be produced from the catabolism of protein. Because the failing kidney does not produce enough NH4+ to excrete all of the fixed acid, metabolic acidosis (with respiratory compensation) results.

32. Which set of arterial blood values describes a patient with chronic renal failure (eating a normal protein diet) and decreased urinary excretion of NH4 +?

33. The answer is E. Untreated diabetes mellitus results in the production of ketoacids, which are fixed acids that cause metabolic acidosis. Urinary excretion of NH4+ is increased in this patient because an adaptive increase in renal NH3 synthesis has occurred in response to the metabolic acidosis.

33. Which set of arterial blood values describes a patient with untreated diabetes mellitus and increased urinary excretion of NH4 +?

When blood flow reaches artery and arteriole how many mmHgs is it?

50

4. The answer is D. The decreased arterial [HCO 3-] is caused by gastrointestinal (GI) loss of HCO3- from diarrhea, not by buffering of excess H+ by HCO3-. The woman is hyperventilating as respiratory compensation for metabolic acidosis.Her hypokalemia cannot be the result of the exchange of intracellular H+ for extracellular K+, because she has an increase in extracellular H+, which would drive the exchange in the other direction. Her circulating levels of aldosterone would be increased as a result of extracellular fluid (ECF) volume contraction, which leads to increased K+ secretion by the distal tubule and hypokalemia.

4. Which of the following statements about this patient is correct?(A) She is hypoventilating(B) The decreased arterial [HCO3-] is a result of buffering of excess H+ by HCO3-(C) The decreased blood [K+] is a result of exchange of intracellular H+ for extracellular K+(D) The decreased blood [K+] is a result of increased circulating levels of aldosterone(E) The decreased blood [K+] is a result of decreased circulating levels of antidiuretic hormone (ADH)

in a capillary, P(c) is 30 mm Hg, P(1) is 2mm Hg, x(c) is 25mm Hg, and x(1) is 2mm Hg, if K(t) is 0.5mL/min/mm Hg, what is the rate of water flow across the capillary wall?

4.50mL/min

40. The answer is C. Once inulin is filtered, it is neither reabsorbed nor secreted. Thus, 100% of the filtered inulin remains in tubular fluid at each nephron site and in the final urine.

40. Which curve describes the inulin profile along the nephron?(A) Curve A(B) Curve B(C) Curve C(D) Curve D

How far is the esophagus from the incisors?

40cm

41. The answer is A. Alanine, like glucose, is avidly reabsorbed in the early proximal tubule by a Na+-amino acid cotransporter. Thus, the percentage of the filtered load of alanine remaining in the tubular fluid declines rapidly along the proximal tubule as alanine is reabsorbed into the blood.

41. Which curve describes the alanine profile along the nephron?(A) Curve A(B) Curve B(C) Curve C(D) Curve D

normal women hematocrit

42 + or - 5 (37-47%)

42. The answer is D. Para-aminohippuric acid (PAH) is an organic acid that is filtered and subsequently secreted by the proximal tubule. The secretion process adds PAH to the tubular fluid; therefore, the amount that is present at the end of the proximal tubule is greater than the amount that was present in Bowman's space.

42. Which curve describes the paraaminohippuric acid (PAH) profile along the nephron?(A) Curve A(B) Curve B(C) Curve C(D) Curve D

43. The answer is B. Alkalinization of the urine converts more salicylic acid to its A- form. The A - form is charged and cannot back-diffuse from urine to blood. Therefore, it is trapped in the urine and excreted.

43. A person who takes an aspirin (salicylic acid) overdose is treated in the emergency room. The treatment produces a change in urine pH that increases the excretion of salicylic acid. What was the change in urine pH, and what is the mechanism of increased salicylic acid excretion?(A) Acidification, which converts salicylic acid to its HA form(B) Alkalinization, which converts salicylic acid to its A- form(C) Acidification, which converts salicylic acid to its A- form(D) Alkalinization, which converts salicylic acid to its HA form

normal male hematocrit

47 + or - 5 (42-52)

How much blood does skin receive

5%

How much blood does the coronary receive

5%

Men normally have how much blood in the body?

5-6 L

if the heart rate is 70bpm then the cardiac output of this ventricle is closest to

5.25L/min

Sickle Cell disease (SCD)

A group of genetic blood diseases, most common is sickle cell anemia. The production of an abnormal hemoglobin called "hemoglobin-S" (also abnormal hemoglobin-C). Normal red blood cells contain hemoglobin-A. Abnormalities in the folding of hemoglobin-S inhibit its ability to pick up and carry oxygen. Buildup of hemoglobin-S in red blood cells causes them to take a "sickle" shape, resulting in the inability to pass through smaller blood vessels and capillaries. Hemoglobin-S can also form aggregates (clusters) in the blood stream. Sickle cells are destroyed rapidly. Signs and symptoms: Hypoxemia (irritates sensory nerves and affects cellular respiration), pain (associated with inflammation and antibody accumulation in joints and abdominal cavity), headaches, fever, fatigue, increased risk of bacterial infection, increased heart rate (tachycardic palpitation due to heart compensating for O2 deficiency), jaundice (damage to erythrocytes causes excessive production of bilirubin) Treatment: supplemental iron, B12, folate, antibiotics for bacterial infection.

The liver stores vitamins ____, _____, and _____.

A, D, B12

What increases urea permeability?

ADH

Collecting tubule has receptor for what?

ADH and aldosterone

What system regulates arterial resistance?

ANS(automonic nervous system

What is the papillary muscles connected to?

AV (atrioventricular valve) by white, cord like substance called chordae tendineae

Excitability

Ability of cardiac cells to initiate action potential in response to inward depolarzing current. Reflects the recovery of channels that carry the inward current for the upstroke of the action potential. Changes over the course of action potential, described by refractory periods: Absolute refractory period, effective refractory period, relative refractory period.

Atrial fibrilation

Abnormal heart rhythm. Similar to atrial flutter. Many atrial foci are rapidly firing. No distinguishable P waves, because atria are sending impulses erratically. Depolarizations from foci near the AV node conduct to the ventricle producing a variable and irregular ventricle response. Irregular. Blood is not completely pumped out and can form a clot. Causes: Different types of heart disease, stress, anxiety, caffeine, alcohol, tobacco, diet pills, open heart surgery, inflammation of heart covering (pericarditis), chest trauma, pulmonary disease, certain medication.

Sickle cell anemia is due to what?

Abnormal hemoglobin

What is atrial tachycardia?

Abnormal movements

Ectopic foci

Abnormal pacemaker sites within the heart (outside of the SA node) that display automaticity. Nearly always result in impulses being conducted over slower pathways within the heart. Thus increasing the time for depolarization and the QRS complex. Can occur within atria or ventricles.

Thrombocytopenia

Abnormally low concentration of platelets

What is the end-diastolic volume?

About 140 mL

What is the pressure when blood reaches the venous system?

About 4 mmHg

What are alpha intercalating cells for?

Absorption of K+ and H+ ions

Atherosclerosis

Accumulation of cholesterol/fat/lipids with connective tissue composed of collagen, elastic fibers, smooth muscle cells, calcium deposits and some inflammatory cells like macrophages and T-cells. Patchy intimal plaques or atheroma. Under the endothelial layer of blood vessel walls of medium and large arteries. Arteries are blocked by fats and cholesterol. Calcium accumulation causes calcification (hardening). Calcification can weaken blood vessels, making them susceptible to rupture, resulting in localized bleeding. If it doesn't rupture, may obstruct blood vessel over time. Hypercholesterolemia, thyroid issues (parathyroid can secrete parathyroid hormone (PTH) in excess), smoking, and drinking may increase risk. -Accumulation of fatty plaque under the endothelial layer activates growth factors involved in cell proliferation and growth attracts white blood cells (neutrophils) which cause inflammation and creates accumulation of calcium. Common arteries affected are: cartoid artery, cerebral artery, coronary artery, aorta and branches, and major arteries located in limbs. If condition worsens, may result in stroke, muscle damage, blood vessel damage and organ damage. Causes: Diabetes, chronic cigarette use, familiar predisposition, hypercholesterolemia or hyperlipidemia, sedentary lifestyle, obesity, and hypertension, dyslipidemia, atherosclerotic plaque. Symptoms: Tight feeling chest pains, shortness of breath. Obstruction can cause Angina (deficiency in blood flow and supply, causing pain). Stroke, muscle damage.

Vagal stimulation of parietal secretion in the direct pathway uses the neurotransmitter _____.

Acetylcholine

which of the following agents or changes has a negative inotropic effect on the heart?

Acetylcholine Ach

Secondary hypertension

Can affect multiple organ systems even during pregnancy. Cause is known and can be treated. Conditions that affect kidneys, arteries, heart, or endocrine system. Coarctation of the aorta, a birth defect in which the aorta is narrowed. Preeclampsia, a condition related to pregnancy, endothelial dysfunction in the maternal blood vessels. Thyroid and parathyroid problems Causes: Adrenal gland disorders: Adrenal cortex secretes aldosterone, cortisol and androgen while adrenal medulla secretes adrenaline and noradrenaline. Can be caused by a tumor, infection or auto-immune disorder, resulting in increased blood pressure. Cushings syndrome (overproduction of cortisol). Hyperaldosteronism (excess aldosterone). Pheochromocytoma (tumor that causes over secretion of hormones like adrenaline and NA). Kidney disease: polycystic kidney disorder, kidney tumor, kidney failure, or narrow/blocked main artery supplying the kidney. Treatment: Corticosteroids (ex: prednisone; anti-inflammatory drug) leads to increased systolic and diastolic pressure and arterial resistance. Non-steroid anti-inflammatory (Motrin, Aleve), weight loss drugs (Meridia). Salt and water retention. Sympathetic activity. Loss of renal vasodilation.

Ventricular premature contraction/complex (VPC or PVC) treatment

Can be resolved by restoring the balance of magnesium, calcium and potassium. -Pharmacological agents: -Class 1: Grouped by the effect they have on sodium channel, and what effect they have on cardiac action potentials. Sodium channel blockers. Lidocaine, phenytoin. -Class 2: Beta blockers. Block the effects of catecholamines at the beta-1 adrenergic receptors, decreasing sympathetic activity on the heart. Decrease conduction through the AV node. Atenolol, propanlol, metroprolol. -Class 3: Blocks potassium channels. Prolonging repolarization. Conduction velocity is not decreased. Sotalol. -Class 4: Caclium channel blocker. Decrease conduction through AV node and shortens phase 2 (plateau) of the cardiac action potential. Reduce contractility of the heart. Allows the body to retain adrenergic control of heart rate and contractility. Verapamil, diltiazem.

Diet pills

Can destroy walls of blood vessels

Velocity is higher in aorta than in what?

Capillaries

What has more intersectional space, arteries or capillaries?

Capillaries

Capillaries lack what kind of muscle and what do they contain?

Capillaries lack smooth muscle and contain pores for gas exchange

Microcirculation/lymph

Capillary bed structure: At junction is a smooth muscle band (precapillary sphincter). True capillaries dont have smooth muscle. Consist of single endothelial layer surrounded by basement membrane. Clefts (pores) allow passage of water soluble substances (small fraction <0.1%). Blood flow is regulated by contraction and relaxation of arterioles and precapillary sphincters. Passage of substances: O2, CO2 (lipid soluble), water, glucose, amino acid (small water soluble), large water soluble substances can cross by pinocytosis. Protein molecules are too large. In brain: clefts are exceptionally tight (blood-brain barrier). Liver and intestine: exceptionally wide and allow passage of protein. Called sinusoids.

Capillaries

Capillary bed: network of capillaries running through tissues. Low permeability capillaries: blood-brain barrier, highly selective only vital substances pass through. Not a barrier against O2, CO2.

What kind of layer is on the outside of the kidney?

Capsule (thin outside layer)

Sometime, we need to block ACE using ACE inhibitor

Captopril

The ______ is where the esophagus enters the stomach.

Cardia

What part of the stomach is this?

Cardia

What part of the stomach is this?

Cardiac notch

What is the equation of cardiac output used in clinics?

Cardiac output = Oxygen consumption/(pulmonary vein oxygen level - pulmonary artery oxygen level)

Circuitry

Cardiac output distribution: -Cerebral: 15% -Coronary: 5% -Renal: 25% -Gastrointestinal: 25% -Skin: 5% -Skeletal muscle: 25%

What is the function of hemoglobin?

Carries oxygen molecule

Arteries

Carry blood away from the heart. Delivery oxygenated blood to tissues. Thick wall. Extensive elastic tissue and smooth muscle (under control of ANS- sympathetic system). Under high pressure. Stressed volume. Little bit less amount of blood than venous system because of thick walls. Arterial pressure is pulsatile, not constant during cardiac cycle. Arterial pressure regulation: Baroreceptor reflex: Fast mechanism (neural), a negative feedback system that is responsible for the minute to minute regulation. Stretch receptors within the walls of cartoid sinus near bifurcation of common cartoid arteries. Will have greatest response, located in aortic arch (respond to increases) Slow mechanism: renin-angiotensin-aldosterone Aorta pressure: 100 mmHg Alpha-1: vasocontriction. Controls blood pressure.

Arteries

Carry blood away from the heart. High pressure. Stressesd volume. Elastic arteries: Largest arteries, high elastin. Diameter range 2.5-1 cm (aorta and branches). Also called conducting arteries. Muscular (distributing) arteries, arterioles (smallest).

Veins

Carry blood back to the heart. Unstressed volume. Low pressure. Smallest veins called venules (small venules called postcapillary venules, merge to form vein). Skeletal muscle pump: muscle presses the veins and pushes blood toward the heart vascular anastomes, vasa corum.

Veins

Carry blood to the heart. Progressively merge to form larger veins. Vena cava (largest vein) returns blood to the heart. Low pressure. Thin wall. Unstressed volume. Contains highest portion of blood. Have alpha-1 adrenergic receptor. Blood flow due to skeletal muscle contraction and having a valve inside the vein to show direction of blood flow. Pressure: 4 mmHg

What region of the stomach includes the antrum and distal body?

Caudad region

Secondary Glomerulonephritis

Cause is more widespread and affects multiple areas in the body, causing many additional disorders.

Birth control pills (combination of estrogen and progesterone)

Causes hypertension in long term

What are basophils for?

Causes the release of histamine durning inflammation

Leukemia

Form of cancer affecting bone marrow, resulting in abnormal leukocyte production. Immature or abnormal leukocytes are not functioning and compromise immune system function. Increasing the risk of infection. Acute: Rapid production of excess immature leukocytes, most common in children. Chronic: Excessive buildup of immature and abnormal leukocytes. Lymphoblastic: "Lymphocytic". Characterized by excessive production of mature lymphocytes which affects the immune system Myeloblastic: "Myelogenous". Characterized by excessive production of both mature and immature myelocytes (a precursor cell to the granulocytes) Symptoms: Inflammation, weight loss, fever, weakness, headaches, bone/joint pain, bleeding easily, swollen lymph nodes in the throat and armpits. Treatment: Chemotherapy: kills leukemic cells. Interferon-alpha (IFNa) growth factor that controls leukemic cell proliferation by slowing it down and increasing the immune system's response for anti-leukemic activity. Radiation therapy: Controls cancerous leukemic cell growth via high energy radiation exposure Stem-cell transplantation (SCT): bone marrow transplant induces synthesis of new, mature and healthy leukocytes. Allows the patient to continue radiation and chemotherapy treatment to get rid of remaining leukemic cells.

Erythropoiesis

Formaiton of erythrocytes

2nd function of yolk sac?

Formation of germ cells (XX for female; XY for male)

Hematopoiesis

Formation of new blood cells. Yolk sac has 2 functions: production of germ cells (XX or XY) which migrate to primitive genitals; formation of stem cells for blood which gives nutrients to embryo because that time of pregnancy there is no complete for of placenta exactly during 1st trimester, after that yolk sac degenerates and placenta is formed, giving nutrients to fetus after formation of fetal liver, liver takes over production of stem cells after birth, newborn, bone marrow is main part for releasing of mother cells or blood cells

Kidney stones

Formation of stones is due to 3 thing: Hypercalcemia due to hyperparathyroidism When patient has hyperparathyroidism, they are at risk for kidney stones because of high level of Ca2+ and calcification Deficiency of a substance called citrate Citrate is a natural Ca2+ channel regulator General hypercalcemia - Excess Ca2+ in blood, there is risk for stones

Venules

Formed from merged capillaries. Pressure: 4 mmHg

Velocity of blood flow

Formula: V= Q/A. Depends on diameter of blood vessels and inter-sectional area which exists in capillary. Directly proportional to blood flow and inversely proportional to cross sectional area.

Ejection fraction

Fraction of the end-diastolic volume ejected in each stroke volume. Related to contractility. Normally 0.55 or 55%. EF= stroke volume/end-diastolic volume

Know the anatomy of the heart

From right to left: Superior/inferior vena cava, arch of aorta, pulmonary trunk/artery, and four pulmonary veins Heart contains four chambers/parts: two atria and two ventricles Between atrium and ventricle, there is the atrioventricular (AV) valve On left side, it is called mitral/bicuspid valve On right side, it is called tricuspid valve

ST segment

From the S wave to the beginning of the T wave. Period when ventricles are completely depolarized. Important in diagnosis of ventricular ischemia or hypoxia (can become either depressed or elevated)

The surgical procedure performed to correct GERD/hiatal hernias is called _______.

Fundoplication

What part of the stomach is highlighted?

Fundus

During fundoplication, the _______ is wrapped around the lower ______ and secured below the _______.

Fundus, esophagus, diaphragm

Gastrin is secreted by the _______ cells of the ________.

G, stomach

Oral ingestion of glucose induces secretion of which hormone?

GIP

This GI hormone is homologous to secretin and glucagon.

GIP

______ is the hormone that is secreted to reduce GI motility when the upper small intestine is already full.

GIP

________ is the only hormone stimulated in response to fat, protein, and carbs.

GIP

What are the 4 main GI hormones?

GIP, CCK, secretin, gastrin

___________ is the candidate hormone binds to beta cells of the pancreas and stimulates insulin secretion.

GLP-1

Vagal stimulation of parietal secretion in the indirect pathway uses the neurotransmitter _____.

GRP

________ is released by vagus nerves and stimulates gastrin release.

GRP

Precipitation of bile salts and cholesterol leads to the formation of ________.

Gall stones

Due to pulmonary edema, normal what cant occur?

Gas exchange. Will lead to death

What passes through the pores of capillaries? What cant pass through?

Gas, small molecules, small proteins, and H2O can pass through Large molecules

This phase of gastric secretion results in 65% of the acid secretion associated with a meal.

Gastric

Gastrin induces growth of the ________ _________.

Gastric mucosa

Caused by a loss of tone in the LES and the reflux of stomach contents into the esophagus.

Gastric reflux

The actions of this hormone are inhibited by somatostatin.

Gastrin

The distention of the stomach stimulates the release of which GI tract hormone?

Gastrin

The gastroileal reflex is mediated by the hormone _______.

Gastrin

This hormone results in the increased secretion of H+

Gastrin

Mitral stenosis

Heart valve disorder. Valve does not open fully, restricting blood flow. Causes: Valve area becomes smaller, less blood flow to body, upper heart chamber swells as pressures builds up, blood may flow back to lungs (pulmonary edema), rheumatic fever, congenital mitral stenosis. Atrial fib (no P waves visual, rhythm is irregular)/flutter, blood clots to brain (stroke), intestines, kidneys, heart failure, pulmonary hypertension (right ventricular hypertrophy (increase in volume) may be seen). Symptoms: atrial fibrilation, chest discomfort (increase with activity, decrease with rest), radiates to arm, chest neck, jaw, or other areas. Tight crushing pressure, cough, possibly bloody (hemoptysis), difficulty breathing during or after exercise or when laying flat, fatigue, tired easily, bronchitis, palpitations, swelling of feet/ankles. Treatment: Cardiac glycosides: alter electrophysiologic mechanisms responsible for arrhythmias. Digoxin (Lanoxin): negatively chronotropic, slowing heart rate by decreasing conduction of electrical impulses through the AV node, controlling heart rate during atrial fibrillation or atrial flutter. Positive intropic: increasing the force of heart rate contraction via inhibition of sodium potassium ATPase pump . Diuretics, beta blockers, calcium channel blockers, anticoagulants, balloon valvotomy, surgical commissurotomy, valve replincreasing the force acement.

Gastric reflux may cause ________.

Heartburn

A major causative factor in the development of gastric ulcers in the presence of _______ ________.

Helicobacter pylori

Surgical treatment of gastric ulcers involves a procedure called a __________.

Hemigastrectomy

A hereditary disease causing the accumulation of iron in the body and liver failure.

Hemochromatosis

A patient with sickle cell complains to you of jaundice. The patient's urine is dark and foamy. What type of jaundice is occurring?

Hemolytic

The _____ ______ and ______ ______ supply the liver with blood.

Hepatic artery, portal vein

Bile drains into the _____ ______ and is stored in the _________.

Hepatic ducts, gallbladder

Bile is produced continuously by ______.

Hepatocytes

Protrusion of part of the stomach into the mediastinum through the esophageal hiatus of the diaphragm.

Hiatal hernia

Hypertension

High blood pressure. Primary and secondary. Symptoms: Usually asymptomatic. Headache, fatigue, shortness of breath, dizziness, convulsion, changes in vision (blurred, double vision), nausea, vomiting, anxiety, increased sweating, nose bleeds, tinnitus (ringing/buzzing in ears), heart palpitations, general feeling of unwellness, increased urination, flushed face, and pale skin.

Dietary Potassium

High potassium diet increases potassium secretion, intracellular increases driving force for potassium secretion. Low diet will decrease. Alpha intercalated discs are stimulated to reabsorb potassium by hyodrgen-potassium ATPase.

Secreted by mast cells of the gastric mucosa, increases gastric H+ secretion.

Histamine

What are the two paracrines released by the endocrine cells of the GI tract?

Histamine, Somatostatin

Blood regulates what?

Hormones, body temperature

Acid-Base

Hydrogen and Potassium exchange across basolateral membrane. -Acidosis decreases potassium secretion: blood contains excess hydrogen. Hydrogen enters across basolateral membrane and potassium leaves the cell. -Alkalosis: Increases potassium secretion: blood contains too little hydrogen. Hydrogen leaves cell across basolateral membrane and potassium enters the cell.

What are some factors that increase contractility?

Hypercalcemia itself can increase heart rate Sympathetic beta-1 (Overstimulation of beta-1 can increase contractility) Digitalis (same thing as digoxin) increases contractility

Preeclampsia

Hypertension during pregnancy. Due to malformed or dysfunctional endothelial layer in maternal blood vessels (those developed to supply maternal blood to the placenta and developing fetus). Changes in maternal hormone levels during pregnancy can cause this resulting damaged blood vessels and increased blood pressure. May be life threatening to mother and child.

Upon examination of a patient's stomach, you see exaggerated folds of rugae. Parietal cell atrophy is evident upon deep mucosal biopsy. What is the correct diagnosis?

Hypertrophic gastritis/ Menetrier's disease

When patient has deficiency of oxygen supply to tissue and cells it leads to what?

Hypoxemia (deficiency of oxygen in blood)

Sickle cell disease has what?

Hypoxemia due to damaged RBCs because of the abnormal hemoglobin

CCK is secreted by the _______ cells of the ________.

I, duodenum, jejunum

The second messenger in parasympathetic stimulation of saliva production is _______.

IP3

UPDATE: Gastrin mediated H+ secretion by parietal cells operates through _____ or ______. (Slides say unidentified)

IP3/Ca2+

Treatment for Cushings syndrome

If case is tumor, they must remove it. If there is infection, they must treat the infection

Series resistance

Illustrated by arrangement of blood vessels within a given organ. Length of blood vessels and viscosoty (concentration) of blood (higher concentration means more viscosity and higher resistance).

Parasympathetic effects on heart rate

SA node, atria and AV node have a parasympathetic vagal innervation, but the ventricles do not. Neurotransmitter is Ach. Acts as muscarinic receptors. Decreases heart rate, conduction velocity (AV node), and contractility (atria only). Vasoconstriction (coronary arteries). -Negative chronotropic effect: Decreases heart rate by decrease the rate of phase 4 depolarization. Fewer action potentials occur per unit time because the threshold potential is reached more slowly and less frequently. Mechanism is decreased sodium inward current. -Negative dromotropic effect: Decreases conduction velocity through AV node. Action potentials are conducted more slowly from atria to ventricles. Increases PR interval. Mechanism is decreased inward calcium current and increases potassium outward current. -Negative intropism: Via muscarinic receptors. Decreasing the inward calcium current during the cardiac plateau.

the ventricles are completely depolarized during which isoelectric portion of the ECG?

ST segment

Cardiac output equation

SV x heart rate (5L = 70L x 70 bpm)

Treatment of pulmonary regurgitation

Same as mitral stenosis Surgery to replace valve

Contractile cells

Sarcomere is the contractile unit. Only have one nucleus. Mitochondria are more numerous (takes up 1/3 volume). Efficient in extracting oxygen (~80%) from passing blood. Shorter, branched, joined together by intercalated discs. Contain tight junctions for binding and gap junctions to allow movement of ions and ion currents between cells. Can conduct action potentials between cell to cell without nerves.

H+ in the duodenum causes the secretion of which hormone?

Secretin

The duodenum secretes _______ due to high acid and fatty acids in it's lumen.

Secretin

This GI hormone is homologous to glucagon.

Secretin

What causes low blood pressure?

Secretion of renin from kidney → renin converts angiotensinogen (comes from liver) into angiotensin I (AgI) → angiotensin converting enzyme (from lung) converts AgI into angiotensin II (AgII) → AgII acts as vasoconstrictor and stimulates secretion of aldosterone from adrenal cortex → aldosterone is carried by blood to nephron (kidney)

Secondary

Side job is to release endocrine hormones (kidney, skin)

Signs of increased androgen in females?

Signs/symptoms: Irregular menstruation and ovulation

What kind of infection can cause endocarditis?

Simple bacterial infection so always take antibiotics for 7-10 days

Type of hiatal hernia where the cardia and part of the fundus move through the esophageal hiatus. Accompanied by reflux and regurgitation.

Sliding hiatus hernia

Effective Refractory Period (ERP)

Slightly longer than ARP. Period during which conducted action potential cannot be initiated.

Atrial pressure

Slightly lower than venous pressure. Left atrial pressure is estimated by the pulmonary wedge pressure. A catheter inserted into the small branches of the pulmonary artery, making almost direct contact with pulmonary capillaries. Approximately equal to left atrial pressure.

Phase 4 (SA node)

Slow depolarization. Accounts for the pacemaker activity (automaticity). Caused by an increase in sodium conductance, resulting in inward sodium current. If (f stands for funny) is turned on by repolarization of the membrane potential during the preceding action potential

Renin-angiotensin-aldosterone system (RAAS)

Slow, hormonal mechanism. Regulation adjusted by blood volume. Renin is an enzyme. -Angiotensin 1: inactive -Angiotensin 2: Physiologically active. Has four effects: stimulates synthesis and sectrion of aldosterone (increases sodium absorption), increases sodium-hydrogen exchange in proximal convoluted tubule (directly increases sodium absorption) leading to contractile alkalosis, increase thirst, causes vasoconstriction of the arterioles (increasing TPR and arterial pressure) -Angiotensin 3: degraded by angiotensinase ex: RAA response to acute blood loss.

Arterioles

Smallest branches of arteries. Site of high resistance. Have smooth muscle wall that is extensively innervated by autonomic nervous fibers. Resistance is regulated by ANS. Largest decrease in pressure occurs across arterioles. Site of highest resistance. Pressure: 50 mmHg (same as artery) Alpha-1 adrenergic receptor found on arterioles of skin, splanchic and renal circulations Beta-1 adrenergic receptor found on arterioles of skeletal muscle.

Causes of atherosclerosis?

Smoking Hypercalcemia Genetics Diabetes (type I and II) Obesity Hypertension

What are some outside factors that can cause arrhythmia?

Smoking, alcohol, and drugs

Aorta, artery, veins and arteriole contain what kind of muscle?

Smooth muscle

NaCl Regulation

Sodium is reabsorbed along the entire nephron, and very little is excreted in urine <1% of the filtered load. -Sodium reabsorption along nephron: Proximal tubule: Reabsorbs 2/3 (67%) of filtered sodium and water. Site of glomerulo-tubular balance. Exactly proportional.

Middle/Late proximal tubule features

Sodium is reabsorbed with chloride.

A paracrine that is secreted in response to H+ in the lumen.

Somatostatin

The paracrine that inhibits the secretion of all GI hormones is called _________.

Somatostatin

D cells of the gastric glands in the stomach secrete _____ and ______ ______.

Somatostatin, intrinsic factor

What is the second factor of ventricular extrasystole

Some tissue in ventricle may have spontaneous depolarization, which send signal to different parts of heart

What is the principal cell for?

Specific cell for ADH and aldosterone reabsorption

The distal part of the esophagus may act as a _______.

Sphincter

Block aldosterone with aldosterone receptor blocker

Spironolactone

Know how blood circulates through the heart

Superior and inferior vena cava collect deoxygenated blood from different parts of the body and then they release their contents into right atrium Right atrium contracts and releases its contents into right ventricle through tricuspid valve Right ventricle contracts and releases its contents into pulmonary artery Pulmonary artery carries deoxygenated blood to lung tissue Lung tissue has gas exchange and releases oxygen molecule into pulmonary capillary Pulmonary capillary becomes four pulmonary veins (2 from right lung, other 2 from left lung) Four pulmonary veins carry fresh oxygenated blood and releases it into left atrium Left atrium contracts and releases fresh blood into left ventricle through bicuspid (mitral) valve Left ventricle contracts and ejects large amount of blood into aorta and aorta carries fresh oxygenated blood to different parts of the body (head and neck, upper/lower limb, etc.)

The duodenum has four parts: _______, _______, _______, _______.

Superior, descending, horizontal, ascending

Increased androgen secretion in females lead to

Suppressed female sex hormones, causing facial acne and hair growth

Treatment for coarctation

Surgery to remove narrow part and connect upper part and lower part

The _________ nervous system is usually inhibitory on the functions of the GI tract.

Sympathetic

How does the ANS control the cardiovascular system?

Sympathetic by beta-1, which has a positive effect

Which division of the ANS conveys visceral (stomach) pain?

Sympathetic nervous system

What system and receptors are stimulated by the information from the baroreceptors?

Sympathetic system and alpha-1 adrenergic receptor

Hyperpolarization of the GI smooth muscle potential is caused by input from the __________ nervous system through the neurotransmitters ______ and ______.

Sympathetic, epinephrine, norepinephrine

What are the two types of circulation?

Systemic and pulmonary circulation

Lymphoid

T and B lymphocytes.

U wave is a continuation of the

T wave

Cortisol increases

TPR and systolic/diastolic pressures

What case is associated with abnormal QT interval

Tachyarrhythmia

What are the steps for last stage of treatment for leukemia?

Take stem mother cell from healthy person's bone marrow and transplant it to patient's bone marrow Those healthy stem cells start to produce healthy WBCs and RBCs

Males have testosterone secretion by what?

Testicles

What is the end result of the high blood osmolality?

The ADH secretion is to prevent urination, to inhibits dehydration, and fluid retention

U wave

is not clear and only sometimes shows up on EKG

Conns disease (hyperaldosteronism)

is specific tumor of aldosterone-producing cells

The other function of aldosterone

is the excretion of potassium into urine

during which phase of the cardiac cycle does the mitral valve open?

isovolumetric ventricular relaxation

during which phase of the cardiac cycle is ventricular volume lowest?

isovolumetric ventricular relaxation

Smalles arteries

arterioles

Beta 1 adrenregic receptors are where?

arterioles of skeletal muscle

Alpha 1 adrenergic receptor accept NE and are located...

arterioles of the skin, splanchnic and renal circulations

When pressure increases during ventricular ejection what does the blood push against and what happens?

The blood pushes against the semilunar valves of aorta and pulmonary arteries, causing the ejection of the stoke volume

Pulmonary circulation is a connection between what

The heart and the lungs

Atrial septum defect

The left and right atrium will have a hole between the two causing the deoxygenated blood from the right atrium to mix with the oxygenated blood from the left atrium.

What takes over blood formation after the yolk sac?

The liver in the fetus

What can inflammation accelerate?

The obstruction of blood vessels

Why is it a problem if the hole from the ductus arteriosus doesn't close?

The oxygenated blood from the aorta can mix with the deoxygenated blood from the pulmonary arteries

What is afterload?

The pressure that exists inside the aorta and pulmonary artery, which come by ventricular contraction

At point 3, what remains in the ventricle?

The remnant of blood, which is about 70 mL. (End-systolic volume)

Chronic Glomerulonephritis

The result of a form of genetic disorder, hereditary nephritis, but in other forms of chronic glomerulonephritis, the cause is unknown.

AV node action potential

The result of inward calcium current.

The ampulla of Vater is surrounded by what?

The sphincter of Oddi

Where are the dead cells taken in?

The spleen

What is systemic circulation?

The superior and inferior vena cava carries blood from different parts of body to heart and aorta takes blood from heart to different parts of the body

Conduction velocity

The time required for excitation to spread throughout cardiac tissue. Depends on size of the inward current during upstroke. The larger the inward current, the higher the conduction velocity. Fastest in purkinje system. Slowest in AV node, allowing time for ventricular filling, if increased the filling may be compromised.

Formation of blood stem cells is formed first by what?

The yolk sac

Lymphoid tissues are

at lymph nodes, spleen, tonsils, thymus -only place where lymphocytes originate

Zigzag is a sign for

atrial fibrillation and mitral valve abnormality

The faster pumping of the atrium is called

atrial flutter or fibrillation

a person's ECG has no P wave but has a normal QRS complex and a normal T wave, therefore, his pacemaker is located in the

atrioventricular node (AV)

AV node is a connection between

atrium and ventricle

Other symptoms with sickle cell anemia

attacks of abdominal pain bone pain breathlessness delayed growth and puberty fatigue fever juandice paleness rapid heart rate lower leg ulcers

18. The answer is B [IX D 4; Table 5-8]. First, the acid-base disorder must be diagnosed. Alkaline pH, low PCO2, and low HCO3- are consistent with respiratory alkalosis. In respiratory alkalosis, the [H+] is decreased and less H+ is bound to negatively charged sites on plasma proteins. As a result, more Ca2+ is bound to proteins and, therefore, the ionized [Ca2+] decreases. There is no respiratory compensation for primary respiratory disorders. The patient is hyperventilating, which is the cause of the respiratory alkalosis. Appropriate renal compensation would be decreased reabsorption of HCO3-, which would cause his arterial [HCO3-] to decrease and his blood pH to decrease (become more normal).

Which of the following statements about this patient is most likely to be correct?(A) He is hypoventilating(B) He has decreased ionized [Ca2+] in blood(C) He has almost complete respiratory compensation(D) He has an acid-base disorder caused by overproduction of fixed acid(E) Appropriate renal compensation would cause his arterial [HCO3-] to increase

What cells do bone marrow produce?

White blood cells (WBCs) , RBCs, and platelets

A hereditary disease that causes the body to retain copper.

Wilson's disease

In intrauterine life what is the embryo connected to?

Yolk sac

What is the treatment for ventricular extrasystole if there is a magnesium/potassium deficiency?

You need to replace the magnesium/potassium

Abnormal QT interval can be a

Zigzag

Anemia

a deficiency of red blood cells

Because of thickness of wall, the capacity is

a little bit lower than venous system

capillary bed

a network of capillaries in a tissue or organ

a patient experiences orthostatic hypotension after a sympathectomy, the explanation for this occurrence is

a suppressed response of the baroreceptor mechanism

The internal structures of veins contain

a valve and is surrounded by skeletal muscle

Arrhythmia means

abnormal movement of ventricles (not strong contraction)

How many BPM is sinus tachycardia

about 100 bpm

Supraventricular means

above the ventricles

What is atherosclerosis?

accumulation of cholesterol, lipid, and/or Ca2+

luekemia can be

acute or chornic

Leukemia can either be...

acute or chronic

What does the adrenal medulla secrete?

adrenaline and noradrenaline

Myelogenous leukemia

affects myelocytes

What are the three main plasma proteins?

albumin, globulins, fibrinogen

Na+ is a good signal for what hormonal secretion?

aldosterone

which agent is released or secreted after a hemorrhage and causes an icnrease in renal Na+ reabsorption?

aldosterone

which receptor mediates constriction of arteriolar smooth muscle?

alpha 1 receptors

Saliva contains _____ _____ which helps break down starch and _____ ______ which helps break down triglycerides.

alpha amylase, lingual lipase

What are the 3 types of globulins?

alpha, beta, gamma

What can excess thyroid hormones irritate?

alpha-1 and beta-1 adrenergic receptors

Excess T3/T4 for hypothyroidism stimulates

alpha-1, which leads to hypertension

in the sinoatrial node phase 4 depolarization (pacemaker potential) is attributable to

an increase in Na+ cunductance

an increase in arteriolar resistance without a change in any other component of the cv system will produce

an increase in arterial pressure

Iron deficiency anemia

anemia caused by inadequate iron intake or lack of iron reception in digestion

sickle cell anemia

anemia caused by lack of effeiceint O2 supply cells in sickle cell shape hemoglobin is deformed as a result

Where is the SA node located?

anterior margin of superior vena cava

V5 and V6 detects

anterior wall and lateral wall of left ventricle

Gamma globulins are

antibodies produced by lymphocytes

When blood osmolality is high then what is active?

antidiuretic hormone

Afterload is the increased pressure that exists in the

aorta or pulmonary trunk

Pulmonary artery's content is released into

aorta through ductus arteriosus

Patent ductus arteriosus connects what during intrauterine` life?

arch of aorta and pulmonary artery

Lumen of blood vessels

area occupied by blood cells

Post myocardial infarction may have ectopic foci which can lead to

arrhythmia

Artery is bigger than

arteriole

inspiration "splits" the second heart sound because

the aortic valve closes before the pulmonic valve

if the ejction fraction increases there will be a decrease in:

the end-systolic volume

Absorption into the renal system means

the filtered ions/fluids will come back into the bloodstream

V1 and V2 detects

the function of right ventricle

Deficiency of oxygen to tissue and cells means ... (What should the heart do?)

the heart should pump faster to compensate, which is the reason for increased HR

Cysts destroy

the kidney which leads to abnormal filtration and normal urine production

25% of Na+ is absorbed by what?

the loop of Henle tubule capillary

The blood in veins is unstressed volume due to what?

the pressure is much lower in veins than arterial system

Eventually, due to atrial fibrillation, blood gets into

the pulmonary veins and the pulmonary veins get hypertension

at which site is systolic blood pressure the highest?

the renal artery

V1-V2 detects

the right ventricle

The atrium receives signals from

the ventricles

Since the atrium receives signals from the ventricles

there is a negative P wave

Why don't we need immature leukocytes?

they are not functional and for this reason, patients have weak immune system

the greatest pressure decrease in the circulation occurs across the arterioes because

they have the greatest resistance

Elastic arteries are ___ than other arteries

thicker

The wall of the arterial system is...

thicker than venous system

Since capillary absorbs these minerals

this leads to increase in blood pressure

Where does the blood carry ADH?

to the nephron of kidneys

which of the following parameters is decreased during moderate exercise?

total peripheral resistance (TPR)

veins

towards heart

valves surrounding the right ventricle (carry deoxygenated b lood)

tricuspid and pulmonary semilunar

valves surrounding the right ventricle (carry deoxygenated blood)

tricuspid, RV, then Pulmoary semilunar

in veins the thickest layer is the

tunica externa

3 layers of BVs

tunica intima- simple squamous cell epithelia tunica media- muscle tunica externa- connective tissue

Venous system is not under pressure so it is called

unstressed volume

which of the following is the result of an inward Na+ current?

upstroke of the action potential in Purkinje fibers

Renal pelvis continues with

ureter

Ureter carries final product of urine to

urinary bladder

What is inside tubule?

urine

Hypertension means any factor, disorder, malformation, malfunction that leads to

vasoconstriction

Hypercalcemia causes

vasoconstriction and hypertension

Overstimulation of alpha-1 adrenergic receptor leads to

vasoconstriction of blood vessels

which of the following is an effect of histamine?

vasodilation of the arterioles


Related study sets

Factoring Polynomials: GCF Assignment

View Set

AICE Geography Unit 4 (Population)

View Set

5. Knowledge Check — Risk Management

View Set

CHAPTER 5. How to Form a Business

View Set

Healthcare Simulation Dictionary

View Set

NVCC BIO 141 - Anatomy and Physiology

View Set

Chapter 24: Asepsis and Infection Control

View Set

Marketing management ch. 3 notes

View Set